SAÚDE

Dengue: por que pacientes com sintomas podem testar negativo para o vírus?



Especialista do CEUB explica a influência do período da infecção, do limite de detecção do teste, isolamento viral, enzimas e da temperatura

Febre, dor de cabeça forte, dor atrás dos olhos, vômito, manchas vermelhas na pele com teste negativo para dengue. Este pode ser um cenário possível e até mesmo comum. Mas por que pacientes com sintomas clássicos da doença podem isentar a detecção pelo vírus transmitido para o Aedes aegypti? Gil Amaro, professor de Ciências Biológicas do Centro Universitário de Brasília (CEUB) e mestre em Biologia Molecular explica que fatores podem influenciar a acurácia de um exame positivo indicando a doença.

Confira a entrevista, na íntegra: 
Por que vários pacientes testam negativo mesmo tendo todos os sintomas clássicos?
Gil Amaro: Cada fase da doença tem um exame específico. Os testes para o período dos sintomas são o RT-PCR, o do Antígeno NS1 ou o de Isolamento Viral. Se a pessoa usar os testes de Anticorpo IgG/IgM no período de sintomas, o resultado será negativo, porque esse teste IgG/IgM é para outra fase da doença. O segundo motivo é que todo exame tem um limite de detecção, sensibilidade e especificidade. Essas três propriedades afetam quando o exame “acerta” ou “erra”. As configurações de ciclo, enzimas e temperaturas no teste RT-PCR podem diminuir a especificidade, afetar o limite de detecção e dar falso negativo. Por fim, a parte pré-analítica pode diminuir a qualidade da amostra dificultando a detecção. Laboratórios com certificações e acreditações têm chance maior de acertar o exame.

Como é feito o teste de dengue? Há mais de um? Existe a possibilidade de um falso negativo? Se sim, por que isso acontece?
Gil Amaro: O padrão é procurar o vírus da dengue ou os anticorpos gerados contra ele no sangue. Situações especiais usam outros fluídos do corpo como saliva, sêmen, urina, líquor ou líquido amniótico. Tem mais de um tipo de exame e cada exame funciona bem em uma fase da doença. Como todo teste tem um limite de detecção, uma sensibilidade e uma especificidade, há uma chance pequena de falsos negativos, mesmo usando o teste correto de cada fase da doença. 

Além dos três motivos citados acima e dos fatores pré-analíticos, analíticos e pós-analíticos, até a temperatura de armazenamento do sangue coletado pode afetar o teste. A ciência está em constante melhoria e os testes são otimizados ano a ano para ficarem mais efetivos, acertando quando tem e quando não tem a doença.

Por que alguns pacientes mesmo tendo todos os sintomas clássicos, característicos de dengue, testam negativo para a doença?
Gil Amaro: Isso ocorre por causa das fases da doença e do uso correto dos testes para cada fase. Vale alertar que outros exames, que não medem diretamente o vírus, ajudam no manejo clínico do paciente que tem dengue, mas o teste deu negativo. O conjunto de sinais e sintomas podem determinar a suspeita de dengue mesmo se um teste der resultado negativo. Existem sinais de alarme e gravidade característicos da dengue, além da sazonalidade. É possível determinar dengue pelos testes laboratoriais ou pelo chamado “vínculo clínico-epidemiológico”.

Existe a possibilidade desse paciente que testou negativo ter contraído uma carga viral menor da doença?
Gil Amaro: Sim, é possível. Cada pessoa tem uma constituição genética e fatores ambientais, como nutrição e o histórico de outras doenças, aliados à presença de anticorpos gerada por infecções anteriores do mesmo subtipo de dengue, pode levar à diminuição mais rápida da viremia. Na maioria dos casos, na segunda infecção por dengue do mesmo subtipo, os anticorpos IgG e IgM atacam o vírus rapidamente, sendo o período e a quantidade de viremia menor. 

Em casos raros, a cada nova infecção a saúde da pessoa fica mais comprometida. E, dependendo da saúde no momento da próxima infecção com a dengue, o cenário pode piorar muito. Plaquetas baixas ou imunidade baixa por outras doenças ou medicamentos pode agravar o caso e prolongar a viremia em tempo e em quantidade da carga viral.

Qual a orientação para os pacientes que testaram negativo mas apresentam todos os sintomas da doença?
Gil Amaro: A orientação é procurar as unidades de saúde para avaliação do vínculo-epidemiológico que determina dengue. Outros exames podem determinar a suspeita de dengue e, até mesmo, classificar o grupo de estadiamento (A, B, C ou D), que vai do mais leve até o mais grave. No Brasil existem unidades de saúde especializadas e as tendas para atendimento inicial. Casos graves tem manejo clínico específico e monitoramento constante para evitar falecimento.

É possível que existam ainda mais casos de dengue do que foi notificado por conta dessa testagem negativa?
Gil Amaro: Sim. Além dos casos falso-negativos existem os portadores assintomáticos, que podem transmitir o vírus mesmo sem ter sintomas. Além do período da história natural de toda doença, quando já aconteceu a infecção, quando as alterações fisiológicas e bioquímicas que permitem detectar sinais e sintomas estão baixas. Resumo: quando está bem no início da infecção.

Quais os perigos do autodiagnóstico de dengue ou de um paciente que, por conta do teste negativo, acredita que não contraiu a doença?
Gil Amaro: O autodiagnóstico é perigoso porque toda infecção por dengue gera perda de líquido dos órgãos e essa perda é mais comum que a hemorragia, podendo agravar. O autodiagnóstico não consegue avaliar o corpo no nível que os exames de sangue fazem. Além disso, há grupos de risco: pessoas com situações específicas ou doenças que, se contraírem dengue, tem chance maior de evoluir para casos graves.

Tanto o exame de sangue quanto o exame clínico permitem avaliar a gravidade da doença pelos sinais de hemodinâmicos no início do agravamento, podendo agir para evitar. Enquanto o autodiagnóstico não abarca todas as possibilidades e a doença pode evoluir até causar danos irreparáveis, quando a pessoa perceber que está com sinais críticos, pode ser tarde demais.

Sinais de Alzheimer podem surgir 40 anos antes em quem tem risco familiar


Imagem: iStock

A idade é o principal fator de risco para o Alzheimer e, muitas vezes, seus primeiros sintomas começam a aparecer após 65 anos. Porém, pessoas com Alzheimer familiar (que ocorre quando vários parentes possuem a demência) costumam apresentar sinais da doença bem antes, e um estudo realizado pela Universidade do Arizona (EUA) descobriu que a partir dos 25 anos já podem demonstrar déficit de memória.

Publicado no periódico eLife, o trabalho científico analisou respostas de quase 60 mil pessoas com idade entre 18 e 85 anos no MindCrowd, um dos maiores projetos de avaliação de como funcionam os cérebros saudáveis — que qualquer pessoa pode responder online e atualmente já conta com a participação de mais de 115 mil indivíduos de 150 países.

Ao analisar esses dados, os pesquisadores descobriram que pessoas com histórico familiar de Alzheimer e menos de 65 anos, em média, têm um pior desempenho de aprendizado e memória, quando comparados a pessoas da mesma faixa etária sem parentes com demência. Em alguns casos déficit foi notado em indivíduos com 40 anos de vida a menos que o início típico da doença — ou seja, com idade na casa dos 25 anos.

Os resultados do estudo sugerem que o efeito da história familiar na memória é particularmente pronunciado entre os homens, especialmente àqueles com menor escolaridade, diabetes e portadores de uma mudança genética comum no APOE, gene associado ao risco de Alzheimer.

Segundo os cientistas, como a demência ainda não tem cura e não há forma comprovada de retardar a perda de memória progressiva causada pela doença, conseguir identificar o quanto antes que alguém tem possibilidade de desenvolver o problema pode ajudar que ela se concentre em táticas para prevenir o Alzheimer.

Como prevenir o Alzheimer?
Coma de forma saudável
Priorize o consumo de vegetais, frutas, sementes e peixes, além de limitar açúcar, carboidratos refinados, gordura saturada e trans. Estudos indicam que a dieta mediterrânea pode ser útil para a prevenção das demências. Alguns nutrientes específicos, como ômega 3, a vitamina E e o resveratrol (presente na uva), já foram associados à redução de risco, mas não há evidências de que tomar suplementos com essas substâncias diminua a possibilidade de ter Alzheimer.

Pratique atividades físicas
Além de preservar a saúde como um todo, os exercícios também têm um papel importante para as funções cognitivas.

Tenha hábitos saudáveis
Cuide de eventuais doenças crônicas, como hipertensão, diabetes, depressão e hipercolesterolemia, e vá ao médico regularmente. Durma bem, não fume e evite o consumo de bebidas alcoólicas.

Cultive o lazer e tenha uma vida social ativa
Conversar com os amigos, ter um hob-by, fazer cursos e ter uma vida intelectual rica ajuda a garantir uma reserva cognitiva, o que pode adiar os sintomas se a pessoa ficar doente. (Do UOL VivaBem)

5 dicas para tratar a dor do joelho e evitar lesões



Você sai do treino com o joelho doendo? Sente dor no joelho para sentar, levantar ou subir uma escada? Então essas dicas são para você, seja qual for a causa das suas dores.

O Porquê das Dores nos Joelhos
De acordo com a Pesquisa “Os Pés Brasileiros” feita pela Pés Sem Dor com 21.423 pessoas, 34,3% dos homens e 46,9% das mulheres têm dor nos joelhos, e 87,1% dessas pessoas dizem que a dor é intermitente. Os joelhos participam de praticamente todos os movimentos que fazemos com os membros inferiores, caminhar, correr, saltar, girar, sentar, levantar, subir e descer. Quando isso é feito com repetição ou impacto em excesso os joelhos ficam sobrecarregados, podendo doer, sofrer lesões ou desenvolver patologias.

Obesidade e o desgaste nas articulações também estão entre as causas das dores nos joelhos, o peso do corpo sobrecarrega as estruturas dos joelhos podendo causar dores e até lesões, como a condromalácia e a artrose, como explica Mateus Martinez, diretor de Fisioterapia na Pés Sem Dor, no vídeo “Dores nos Pés e Joelhos: qual a relação com o peso”. E o desgaste nas articulações, embora seja mais comum entre idosos, pode acontecer mais cedo. Além, é claro, dos traumas, acidentes envolvendo os joelhos são comuns e, dependendo do grau, podem exigir até intervenção cirúrgica.

Joelhos Valgos e Varos
As pernas de quem tem joelhos valgos formam um “X”: os joelhos são mais próximos do que os pés. Ao contrário, as pernas de quem tem joelhos Varos formam um “arco”: os joelhos se projetam para fora, distanciando-se um do outro, conta Mateus Martinez no vídeo: Saiba o que é Joelho Valgo e Varo.

Essas alterações podem fazer parte da anatomia da pessoa, ou podem ocorrer em função da forma como a pessoa pisa, se a pisada é pronada, ou seja, para dentro, a tendência é que a pessoa tenha o joelho valgo, e se a pisada é supinada, para fora, o joelho tende a ser varo. Ter joelhos varos ou valgos não significa necessariamente que a pessoa vai sofrer com dores. Porém, se os seus joelhos doem é importante verificar se o formato do joelho ou o jeito de pisar não está na causa do problema, ou não está agravando as dores.

Como combater e prevenir as dores nos joelhos
1. Atividade Física – Além de fortalecer e melhorar a mobilidade dos joelhos, a atividade física ajuda na perda de peso, combatendo as causas da dor. Porém, quem já sofre com dores nos joelhos precisa contar com acompanhamento profissional, um educador físico vai definir um programa de exercícios de baixo impacto: natação, pilates, caminhada e hidroginástica são mais indicados por não sobrecarregar os joelhos.

2. Dieta – Além da perda de peso, que vai reduzir a sobrecarga eliminando a causa das dores, quem tem artrose também deve buscar a orientação de um nutricionista para consumir alimentos antiinflamatórios, que estimulem o aumento da síntese de colágeno e cartilagem, elementos que protegem os ligamentos dos joelhos.

3. Fisioterapia para fortalecimento – Além de exercícios específicos, a fisioterapia pode contribuir com diversos recursos fisioterapêuticos que ajudam na eliminação das dores e no fortalecimento. Quando os músculos da coxa, joelho, panturrilha e outros são fortalecidos, passam a absorver parte do impacto que o joelho sofre no dia a dia, protegendo os ligamentos e cartilagens e reduzindo o desgaste.

4. Palmilhas sob medida para joelho valgo e para joelho varo - As palmilhas ortopédicas, feitas sob medida podem ajudar e muito a pessoa que sofre com dores nos joelhos. Primeiro, porque ela corrige a pisada oferecendo suporte para que os seus pés não pisem nem para fora, nem para dentro, alinhando a pisada e os joelhos, evitando lesões e dores. A palmilha também ajuda a absorver impacto nas atividades do dia a dia, poupando tanto os pés, quanto os tornozelos e os joelhos. Quem pratica atividade física pode contar com palmilhas específicas que vão reduzir o estresse pela repetição e proteger pés e joelhos de movimentos que possam provocar dores e lesões.

5. Consulte um especialista – Não subestime suas dores, se o joelho está incomodando, procure a ajuda de um especialista. A Pés Sem Dor conta com quase 80 unidades de atendimento pelo Brasil onde oferece avaliação gratuita dos pés, tornozelos e joelhos. Durante a avaliação é possível conhecer o tipo de pisada, além de verificar se joelhos Valgos ou Varos estão causando estresse, sobrecarga ou lesões. A Pés Sem Dor também orienta sobre a necessidade de acompanhamento médico e exercícios, além de indicar o tratamento com palmilhas ortopédicas sob medida. Interessados podem agendar a avaliação gratuita pelo site https://www.pessemdor.com.br/agendamento/ ou pelo telefone 4003-8883.

A Pés Sem Dor também fornece mais dicas exclusivas sobre todo tipo de dores, inclusive joelhos, em seu perfil oficial no Instagram.


Ter pés chatos pode ser problema?


Todo mundo nasce com os pés sem curvatura, os famosos “pés chatos” ou planos. Mas quem permanece com o problema ao longo da vida pode sofrer com dores nos pés, tornozelos e joelhos. Nos casos mais críticos a pessoa pode até ter dificuldades para caminhar. Já parou para analisar como são os seus pés?

Pés Normais, Chatos ou Cavos?
Existem 3 tipos de pés: o pé normal tem uma leve curvatura conhecida como arco plantar que amortece o impacto que os pés recebem ao caminhar, correr ou saltar, assim explica Mateus Martinez, diretor de fisioterapia da Pés Sem Dor, empresa renomada na área. 

O pé cavo também tem essa curvatura, mas ela é mais pronunciada. No pé cavo o contato com o chão só ocorre em dois pontos: o calcanhar e a ponta dos pés. Ele também é mas rígido, ou seja, o arco plantar tem menos mobilidade para absorver impacto. 

Por fim, existe o pé chato ou plano. Nele, o arco plantar tem só um pouquinho de curva ou quase nada. Como dizem os profissionais, é um arco “desabado”. O pé chato fica todo apoiado no chão. Não tem amortecimento. 

Segundo pesquisa “Os Pés Brasileiros” realizada pela Pés Sem Dor com 21.423 pessoas, 14% da população tem pés chatos em algum grau. O pé chato tende a aumentar com a idade, pois é comum haver um “desabamento” do arco do pé com a idade, esse processo é conhecido como pé chato “adquirido”. O aumento do peso corporal também contribui para o surgimento do pé chato “adquirido” tanto para homens, quanto para mulheres.

Problemas comuns em quem tem pé chato
Quem tem pés chatos pode sofrer com dores decorrentes de vários problemas. O mais comum é a Fascite Plantar, como conta, Mateus Martinez no vídeo “Pé Chato e Cavo – entenda as diferenças”. A falta de amortecimento expõe a fáscia plantar a muito impacto, podendo gerar um processo inflamatório, a Fascite Plantar. E essa situação é agravada quando a pessoa tem sobrepeso, permanece muito tempo em pé ou pratica atividades físicas de impacto: corrida, vôlei, basquete, enfim, esportes onde os pés são mais exigidos.

Pessoas com pés chatos ou muito chatos também podem ter problemas de dor no tornozelo. E a razão é que o pé chato tende a virar para dentro, ocasionando um desalinhamento no corpo que pode causar dores no tornozelo, na canela e no joelho, explica Mateus. Quem tem pé chato pode sofrer com canelite, por exemplo, ou pode desenvolver uma síndrome femoropatelar, trazendo fraqueza e dores nas articulações dos joelhos.

Como saber o se o seu pé é chato
Primeiro, é importante dizer que nem todo mundo que tem pés chatos está condenado a sofrer com dores. Mas se você tem dor nos pés, tornozelos, canelas ou joelhos é interessante procurar saber se a dor tem relação com o seu tipo de pé. Uma forma de avaliar é observando a marca que os seus pés fazem no chão quando você sai do banho. Se a marca que ficar no chão é bem completa, sem espaços vazios é bem provável que o seu pé seja chato. Nesse caso, o mais indicado é procurar orientação profissional. Na Pés Sem Dor é possível passar por uma avaliação gratuita dos pés e saber qual o tipo do seu pé, como é a sua pisada e qual a relação das dores com essas questões anatômicas e biomecânicas.

Pé chato tem jeito?
Existem exercícios e fisioterapia para ativar o músculo do arco plantar melhorando a curvatura ou impedindo que um pé chato se torne “muito chato” e passe a provocar dores e interferir na forma de caminhar, explica Mateus Martinez no vídeo “Pé Chato – como corrigir”. 

Palmilhas ortopédicas sob medida também costumam ser indicadas no tratamento do pé chato. Como são feitas de acordo com a necessidade específica de cada pessoa, essa palmilha vai proporcionar apoio ao arco plantar, que ajuda a absorver impacto, reduzindo ou eliminando dores. A palmilha sob medida para pés chatos também pode ter uma elevação lateral para ajudar a alinhar o tornozelo melhorando a postura e prevenindo dores no tornozelo, canelas e joelhos, explica Mateus. 

Interessados em fazer uma avaliação gratuita dos pés com um especialista da Pés Sem Dor podem encontrar no site https://www.pessemdor. com.br/agendamento/ a unidade mais próxima de seu endereço e fazer o agendamento. A Pés Sem Dor também atende pelo telefone 4003-8883, de qualquer lugar do Brasil.


O que é o Autismo? 


O Transtorno do Espectro do Autismo (TEA) reúne desordens do desenvolvimento neurológico presentes desde o nascimento ou começo da infância. São elas: Autismo Infantil Precoce, Autismo Infantil, Autismo de Kanner, Autismo de Alto Funcionamento, Autismo Atípico, Transtorno Global do Desenvolvimento sem outra especificação, Transtorno Desintegrativo da Infância e a Síndrome de Asperger.

Segundo o Manual Diagnóstico e Estatístico de Transtornos Mentais DSM-5 (referência mundial de critérios para diagnósticos), pessoas dentro do espectro podem apresentar déficit na comunicação social ou interação social (como nas linguagens verbal ou não verbal e na reciprocidade socioemocional) e padrões restritos e repetitivos de comportamento, como movimentos contínuos, interesses fixos e hipo ou hipersensibilidade a estímulos sensoriais. Todos os pacientes com autismo partilham estas dificuldades, mas cada um deles será afetado em intensidades diferentes, resultando em situações bem particulares. Apesar de ainda ser chamado de autismo infantil, pelo diagnóstico ser comum em crianças e até bebês, os transtornos são condições permanentes que acompanham a pessoa por todas as etapas da vida.

Entenda
As causas do TEA não são totalmente conhecidas, e a pesquisa científica sempre concentrou esforços no estudo da predisposição genética, analisando mutações espontâneas que podem ocorrer no desenvolvimento do feto e a herança genética passada de pais para filhos. No entanto, já há evidências de que as causas hereditárias explicariam apenas metade do risco de desenvolver TEA. Fatores ambientais que impactam o feto, como estresse, infecções, exposição a substâncias tóxicas, complicações durante a gravidez e desequilíbrios metabólicos teriam o mesmo peso na possibilidade de aparecimento do distúrbio.

O TEA afeta o comportamento do indivíduo, e os primeiros sinais podem ser notados em bebês de poucos meses. No geral, uma criança do espectro autista apresenta os seguintes sintomas:

Dificuldade para interagir socialmente, como manter o contato visual, expressão facial, gestos, expressar as próprias emoções e fazer amigos;

Dificuldade na comunicação, optando pelo uso repetitivo da linguagem e bloqueios para começar e manter um diálogo;

Alterações comportamentais, como manias, apego excessivo a rotinas, ações repetitivas, interesse intenso em coisas específicas, dificuldade de imaginação e sensibilidade sensorial (hiper ou hipo).

O Manual Diagnóstico e Estatístico de Transtornos Mentais DSM-5 rotula estes distúrbios como um espectro justamente por se manifestarem em diferentes níveis de intensidade. Uma pessoa diagnosticada como de grau 1 de suporte apresenta prejuízos leves, que podem não a impedir de estudar, trabalhar e se relacionar. Um indivíduo com grau 2 de suporte tem um menor grau de independência e necessita de algum auxílio para desempenhar funções cotidianas, como tomar banho ou preparar a sua refeição. Já o autista com grau 3 de suporte vai manifestar dificuldades graves e costuma precisar de apoio especializado ao longo da vida.

Por outro lado, o diagnóstico de TEA pode ser acompanhado de habilidades impressionantes, como facilidade para aprender visualmente, muita atenção aos detalhes e à exatidão; capacidade de memória acima da média e grande concentração em uma área de interesse específica durante um longo período de tempo.

Cada indivíduo dentro do espectro vai desenvolver o seu conjunto de sintomas variados e características bastante particulares. Tudo isso vai influenciar como cada pessoa se relaciona, se expressa e se comporta.


Já tomou água hoje? Veja os principais benefícios do consumo da bebida para a saúde



Coordenadora de Nutrição e Dietética destaca o papel vital da água no organismo e discute seus benefícios para a saúde e prevenção de doenças

A água desempenha um papel fundamental em nosso organismo, sendo essencial para o bom funcionamento de diversos sistemas e processos vitais.

Mas, apesar de ser muito importante para a saúde, o hábito de beber água ainda é algo que a maioria das pessoas acaba negligenciando.

1. Mais de 70% do corpo é composto de água
Esse líquido participa de praticamente todas as funções do organismo, sendo essencial aos tecidos corporais e fundamental no transporte e na diluição de diversas substâncias, entre macro e micronutrientes. Participa também do processo de digestão, absorção e excreção e auxilia, ainda, a eliminação de toxinas e a filtração renal.

2. Retarda o envelhecimento
Esse efeito está relacionado ao tópico anterior: nossos órgãos são formados por células e nossas células têm água em sua composição. “Portanto, se o organismo não recebe a quantidade necessária de água, essa falta acelera a oxidação celular, causando o envelhecimento”, esclarece Cintya.

3. Previne desidratação
Quando a ingestão de água é insuficiente, o organismo pode apresentar sintomas de desidratação, como sede exagerada, boca e pele seca, olhos fundos, diminuição da sudorese, cansaço, dor de cabeça e tontura.

4. Ajuda na concentração
Um estudo sobre mulheres publicado no The Journal of Nutrition explorou o estado de hidratação na função cerebral e descobriu que uma perda de 1,4% do peso corporal em líquidos durante o exercício causava uma queda na concentração.

5. Regula a temperatura corporal
Nosso corpo remove o líquido através do suor; assim, precisamos estar hidratados a fim de que esse processo seja realizado com eficiência. “Quanto mais quente o ambiente, mais importante é a água. A desidratação em um ambiente quente pode causar insolação”, afirma a especialista.

6. Ajuda na prevenção de pedra nos rins
Sem água suficiente, a produção de urina cai, o que pode permitir que os minerais formadores de pedras se acumulem nos rins e na bexiga. Da mesma forma, consumir líquido suficiente pode ajudar a controlar o risco de infecções do trato urinário (ITU), pois a sub-hidratação pode promover o crescimento de bactérias causadoras de infecção, de acordo com a UCLA Health.

7. Previne constipação
De acordo com especialistas, não beber líquido suficiente é uma causa comum de constipação.

Como consumir água?
Existe uma recomendação geral de aproximadamente 35 ml por cada kg de peso, ou seja, uma pessoa com 65 kg deve tomar aproximadamente 2,27 litros de água por dia, por exemplo. Assim, a quantidade de água necessária está associada a cada condição individual.

De acordo com a coordenadora de nutrição, “podemos hidratar o organismo de duas maneiras: a ingestão de líquidos é a forma mais eficaz e inclui, além da água, sucos naturais, água de coco e chás como camomila, erva-doce e cidreira. A segunda forma ocorre por meio da ingestão de alimentos que possuem água na composição. Entre as opções que proporcionam mais hidratação, estão frutas e legumes, como melancia, morango, pêssego, abobrinha, pepino e tomate”.

Quente ou fria?
Algumas vantagens podem ser observadas ao consumir água em determinadas temperaturas:

Beber água quente: caso esteja gripado, resfriado ou com sintomas de garganta e fadiga, desfrutar de uma bebida quente em uma caneca pode melhorar um pouco seu ânimo. De acordo com pesquisa publicada na Rhinology, uma bebida quente com frutas aliviou um número maior de sintomas de um resfriado comum, comparada a uma bebida de frutas idêntica, servida em temperatura ambiente;

Beber água fria: consumida em temperaturas frias, a água pode trazer maior eficácia na hidratação após um treino. O Jornal Internacional de Medicina Clínica e Experimental publicou um pequeno estudo no qual observou-se que voluntários beberam mais quando a água estava a 16°C e a consumiram significativamente menos quando a 5°C. O objetivo da análise era avaliar se a temperatura da água afeta o quanto uma pessoa a ingere voluntariamente após um grande esforço.

A hidratação é sempre bem-vinda, em especial ao acordar, durante a atividade física, antes e após comer e ao se deitar. Com sua garrafinha cheia, as chances de manter-se saudável e prevenir doenças é muito maior. (Por Fernanda Martinelli)


Dor no calcanhar tem remédio?



Segundo pesquisa realizada pela Pés Sem Dor com 2.940 pessoas, a dor no Calcanhar incomoda mais de 61% dos trabalhadores brasileiros, sendo a principal queixa de dores nos pés entre homens e mulheres.

Sabia que o calcâneo, o osso do calcanhar, é o maior do nosso pé? Ele é também o ponto que mais carga e impacto recebe no dia a dia, por isso tanta gente se queixa de dores na região do calcanhar; assim observa Mateus Martinez, fisioterapeuta chefe da Pés Sem Dor.

Por que o calcanhar dói?
Fascite Plantar – Caso sinta dores agudas no calcanhar pela manhã, logo nos primeiros passos do dia, é quase certo que esteja sofrendo por causa da fascite plantar: uma inflamação ou degeneração na fáscia, um tecido que abrange toda a planta do nosso pé, dos dedos até o calcanhar, explica Mateus Martinez. 

Na medida em que a pessoa anda, ela “massageia” a sola dos pés dispersando o fluido inflamatório e aliviando as dores. Mas se ficar novamente em repouso, o fluido inflamatório volta a se acumular, trazendo aquela dor aguda de volta. A fascite pode ocorrer por vários motivos: problemas na pisada, pés muito cavos ou muito chatos, sobrepeso e alguns tipos de atividade esportiva que podem sobrecarregar a fáscia e causar a inflamação.

Esporão de Calcâneo – A dor no calcanhar também acontece por causa do esporão: uma “ponta” óssea que aparece na base do calcanhar. Felizmente, o esporão nem sempre causa dores, mas ele pode pegar em algum músculo, tecido ou ligamento, e nesses casos, pode sim causar dores. 

O esporão acontece por causa do excesso de impacto no calcanhar: atividade esportiva com movimentos repetitivos, calçados muito rígidos, obesidade, enfim: tudo o que coloca muita carga no calcanhar pode criar condições para o surgimento do esporão, explica Mateus.

Bursite no Calcanhar – A sua dor é na região posterior do calcanhar, perto do Tendão de Aquiles? Então você pode ter bursite: uma inflamação na Bursa do calcanhar. A Bursa é uma bolsa cheia de líquido que funciona como uma “almofada” protetora entre o osso e o Tendão de Aquiles que ajuda a diminuir o atrito e facilita os movimentos, pontua Mateus Martinez. Quando a Bursa inflama, causa dor intensa que piora com a atividade física ou com pressão na região, também pode provocar inchaço e vermelhidão na região afetada. A dor pode ser forte o suficiente para dificultar movimentos, impedindo, em alguns casos, até que a pessoa fique em pé.

Entre as causas da bursite no calcanhar podem estar o impacto e os movimentos repetitivos de algumas atividades esportivas, mas o problema também pode ser resultado de lesões ou pancadas na região do calcanhar, calçados apertados ou inadequados e outras patologias como artrite, gota, esporão de calcâneo e Tendinite de Aquiles.

Proteja o seu calcanhar e resolva as dores com palmilhas ortopédicas sob medida.
Uma das formas mais eficazes de tratar a dor no calcanhar é com o uso de palmilhas ortopédicas. Mas é importante optar sempre por palmilhas sob medida, feitas de acordo com as dimensões dos pés e as necessidades específicas de cada pessoa. A palmilha ortopédica vai dar suporte ao arco plantar, aumentando as áreas de contato dos pés com o calçado, distribuindo melhor o peso do corpo e aliviando os pontos dolorosos, explica Mateus Martinez. A palmilha não apenas resolve a dor, mas evita que as patologias se agravem, além de corrigir a pisada e alinhar a postura.

Como conseguir uma palmilha sob medida
Embora existam vários profissionais e clínicas desenvolvendo palmilhas sob medida de modo artesanal, a solução mais inovadora e eficaz que há no mercado é a palmilha feita em impressora 3D, com precisão digital. Na Pés Sem Dor, além de imprimir as palmilhas com a tecnologia 3D, também é feita uma avaliação gratuita dos pés da pessoa com um especialista, usando Baropodômetro e scanners 3D para um diagnóstico mais preciso e assertivo. Hoje a Pés Sem Dor conta com mais de 70 unidades pelo Brasil, todas com equipamentos de última geração para avaliar os pés dos clientes. As avaliações gratuitas da Pés Sem Dor são feitas com hora marcada. Interessados podem agendar através do site https://www.pessemdor.com.br/agendamento ou pelo telefone 4003-8883, de qualquer lugar do Brasil.


4 dicas para Amenizar o Inchaço nas Pálpebras




Calor, noites mal dormidas e consumo alto de sal podem levar ao inchaço na região palpebral

Embora o verão esteja com os dias contados, o calor ainda predomina na maior parte dos estados brasileiros. E, nos dias mais quentes, podem surgir alguns desconfortos, como o inchaço nas pálpebras.

Isto ocorre porque o calor leva à dilatação dos vasos sanguíneos para ajudar no resfriamento do corpo. Como consequência, há um extravasamento dos líquidos que se acumulam no espaço entre as células. O resultado é o inchaço que costuma afetar mãos, pés, pernas e, em alguns casos, as pálpebras.

Inchaço nas pálpebras tem outros motivos
Mas, segundo Dra. Tatiana Nahas, oftalmologista geral e especialista em cirurgia plástica ocular, não é só o calor que pode levar ao edema na região palpebral. “O inchaço nas pálpebras também pode acontecer devido a alguns comportamentos como maior ingestão de álcool, consumo de alimentos com muito sal e sódio e noites mal dormidas”.

Edema pode ser temporário ou permanente
O inchaço nas pálpebras que surge repentinamente, principalmente na presença destes fatores comportamentais, como ingerir álcool, dormir pouco e consumir alimentos ricos em sódio e sal, tende a melhorar quando a pessoa muda estes hábitos.

Por outro lado, se mesmo adotando comportamentos mais saudáveis, o inchaço permanece, é possível que esta manifestação tenha relação com o processo natural do envelhecimento.

“Ao longo da vida, ocorrem diversas mudanças nas estruturas da região periocular, como enfraquecimento dos músculos, perda do volume ósseo e flacidez da pele. Estas alterações podem levar ao escape da gordura, acarretando as temidas bolsinhas embaixo dos olhos”, comenta Dra. Tatiana.

Portanto, quando a pessoa já apresenta estas bolsinhas e se expõe a outros fatores de risco, o inchaço pode se agravar. Apenas lembrando que em alguns casos, o edema palpebral está associado a doenças sistêmicas, como problemas renais e na tireoide.

4 Dicas para amenizar o inchaço das pálpebras
Veja agora algumas dicas para amenizar o inchaço das pálpebras.

1- Hidrate-se
A água é o melhor líquido para se hidratar. Procure ingerir pelo menos 2 litros por dia. Você também pode investir na água de coco, chás gelados e sucos naturais. Porém, prefira sucos sem adição de açúcar.

2- Controle a ingestão de álcool
O consumo exagerado de bebidas alcoólicas é prejudicial para todo o organismo. Portanto, procure usar o bom senso e o equilíbrio.

Outra dica é que o álcool desidrata o corpo, ou seja, juntamente com a sua bebida alcoólica você deve consumir água, na mesma proporção. Exemplo: para cada taça de vinho, tome também uma taça de água.

3- Cuide da alimentação
Aproveite o calor para consumir mais alimentos naturais, como frutas, verduras e legumes. Nos lanchinhos, procure opções mais saudáveis como castanhas, nozes, iogurtes etc.

Cuidado com o excesso de sal e atente-se aos alimentos ricos em sódio. Lembre-se que até água com gás tem sódio! Leia os rótulos.

4- Aposte nas compressas frias
Uma vez que o calor leva à vasodilatação e suas consequências, como o acúmulo de líquido entre as células, o frio leva à vasoconstrição. Desta maneira, as famosas compressas de água fria nos olhos podem ajudar a amenizar o inchaço nas pálpebras.

“Há várias maneiras de fazer as compressas, como usar saquinhos de chá com água gelada, algodão ou gaze embebido em água gelada ou ainda bolsinhas térmicas específicas para a região ocular. É possível encontrar estas bolsinhas à venda em sites e até mesmo em farmácias. O único cuidado é não colocar gelo diretamente na pele, pois isto pode causar queimaduras”, finaliza Dra. Tatiana.


Estudo indica que alguns fármacos usados para tratar diabetes ajudam a prevenir demência




Apesar das fortes indicações dos efeitos desses fármacos para diabetes na redução do risco de demência, ainda é preciso entender melhor como ocorre esse processo, explica o neuro-ortopedista Dr. Luiz Felipe Carvalho

A diabetes é uma condição crônica que afeta o metabolismo da glicose, gerando níveis elevados de açúcar no sangue. De acordo com a Sociedade Brasileira de Diabetes, cerca de 6,9% da população do Brasil sofre da doença atualmente, o que mostra a necessidade de mais estudos sobre a patologia.

As novas pesquisas sobre a doença têm ajudado a compreender melhor a sua formação e impactos na saúde do paciente, incluindo a sua relação com demências e disfunções cognitivas.

Diabetes e demência: Há causalidade?
O novo estudo “Diabetes e demência: Como alguns fármacos para tratar diabetes reduzem o risco de demência”, publicado na revista científica Cuadernos de Educación y desarrollo pelo Neuro-Ortopedista, Dr. Luiz Felipe Carvalho, em parceria com a especialista em Nutrologia Dra. Rosany de Sales e o Pós PhD em neurociências, Dr. Fabiano de Abreu Agrela, explora a relação entre demência e diabetes.

“Existem diversos estudos que apontam uma relação entre a diabetes e a demência, mostrando também que quanto mais longa for a duração da doença, maiores são os riscos de comprometimento cognitivo.”

“Estudos têm indicado uma relação epidemiológica entre a presença de diabetes tipo 2 e disfunções cognitivas, que podem variar desde um leve comprometimento à demência, com um risco aumentado em relação ao tempo que se tem a doença, impacto que pode ter relação com fatores como a resistência à insulina, hiperglicemia, desregulação energética e neuroinflamação”, explica.

“Além disso, a diabetes tipo 2 gera perdas de memórias em cerca de 10% dos indivíduos que sofrem com a doença e é considerada um risco para demência vascular, doença de Alzheimer, comprometimento cognitivo leve e demência mista, mas ainda é necessário detalhar melhor os fatores envolvidos nessa relação”, ressalta Dr. Luiz Felipe Carvalho.

Impacto dos fármacos antidiabéticos na redução do risco de demência
Ainda segundo o estudo, alguns medicamentos usados tradicionalmente para tratar diabetes também podem ajudar a prevenir a demência, explica Dr. Luiz Felipe Carvalho.

“Um dos fatores mais indicados na relação diabetes-demência é o mal controle glicêmico e episódios de hipoglicemia severa, mas declínios também podem ser notados ainda no estágio pré-diabético.”

“Alguns medicamentos, por exemplo, foram associados à redução do risco de déficits cognitivos, por exemplo, a sulfonilureia, com redução de 12% no risco de demência por todas as causas, mas aumento de 14% do risco de demência vascular em comparação com a monoterapia com  metformina. Enquanto isso, a monoterapia  com  tiazolidinediona apresentou redução de 22% do risco de demência por todas as causas, 11% no risco de Alzheimer e 57% no risco de demência vascular.”

“Os resultados de estudos clínicos sobre medicamentos antidiabéticos e demência são variados, mas ressaltam a necessidade de pesquisas mais amplas e de longo prazo”, explica o Dr. Luiz Felipe Carvalho.


Fevereiro Laranja: leucemia ocupa décima posição entre os tipos de câncer mais frequentes no Brasil


Especialista alerta para identificação precoce da doença

A leucemia é um dos tipos de câncer mais comuns no Brasil. Sem considerar os tumores de pele não-melanoma, ele ocupa a décima posição entre os mais frequentes, segundo o Instituto Nacional de Câncer (Inca). Além disso, o Instituto prevê 11.540 novos casos de leucemia por ano até 2025.

O diagnóstico precoce é fundamental para o sucesso do tratamento e cura da leucemia. E é exatamente por isso que a campanha Fevereiro Laranja faz um alerta para a conscientização sobre a doença e importância da doação de medula óssea para o tratamento do câncer.

A leucemia afeta as células sanguíneas, principalmente os glóbulos brancos, responsáveis pela defesa do organismo contra infecções. Andréia Moraes, hematologista da Pró-Saúde, explica as consequências da doença.

“Como a leucemia afeta a produção de células sanguíneas, o indivíduo pode apresentar anemia, sangramentos e hematomas, devido a diminuição de glóbulos vermelhos ou interferência na produção de plaquetas”, alerta a profissional.

“Com isso, os principais sintomas são fadiga, falta de ar, palpitação, dor de cabeça, fraqueza, palidez, sangramentos na gengiva, manchas roxas na pele e infecções frequentes”, completa. 

Tipos de leucemia
Existem diferentes tipos de leucemia, que são classificados de acordo com o tipo de célula acometida e a maturidade dessas células. Dentre os principais estão:

• Leucemia Linfoide Aguda: mais comum em crianças, apresenta células jovens do tipo linfoide;

• Leucemia Linfoide Crônica: comum entre adultos, principalmente a partir dos 50 anos, seu desenvolvimento é lento, e com predomínio de linfócitos maduros.

• Leucemia Mieloide Aguda: mais frequente em adultos, apresentando células imaturas do tipo mieloide;

• Leucemia Mieloide Crônica: apresenta células mieloides com vários níveis de maturação.

Andréia explica que toda célula passa por um processo de amadurecimento que acontece dentro da medula óssea e cada uma apresenta um desenvolvimento diferente no organismo.

“Nas leucemias agudas, observa-se a presença de glóbulos brancos que não amadurecem, manifestando um crescimento rápido. Enquanto nas leucemias crônicas, apesar de se multiplicarem mais rápido e serem prejudiciais, têm seu desenvolvimento mais lento”, esclarece. 

Como ser um doador de medula óssea
Para se tornar um doador voluntário de medula óssea e ajudar na cura de pacientes com leucemia, é preciso ir ao Hemocentro, realizar um cadastro no REDOME (Registro Brasileiro de Doadores Voluntários de Medula Óssea) e coletar uma amostra de sangue. Além disso, é preciso ter entre 18 e 35 anos, e não ser diagnosticado com nenhuma doença impeditiva, como hepatite, Aids/HIV e outras doenças autoimunes.


Câncer infantil: sintomas não devem ser ignorados 




As chances de cura ficam em torno de 85%, quando a doença é identificada nos estágios iniciais e tratada em centros especializados

O dia 15 de fevereiro é marcado como a data mundial de conscientização do câncer infantojuvenil, um grupo de várias doenças que pode ocorrer em qualquer local do organismo por conta da proliferação descontrolada de células anormais.

E é importante que pais e/ou responsáveis fiquem atentos aos sintomas: palidez, dor óssea, hematomas, caroços, inchaços, dores de cabeça ou nos membros, alterações oculares, febre persistente, náuseas, sudorese noturna, mudanças de visão, cor esbranquiçada atrás da pupila, e perda de peso repentina.

“As chances de cura ficam em torno de 85%, quando o câncer é identificado nos estágios iniciais e tratado em centros especializados”, revela o prof. Dr. Vicente Odone Filho, oncopediatra e diretor clínico do Instituto de Tratamento do Câncer Infantil - ITACI.

Segundo o especialista, os tumores mais comuns são as leucemias, que afetam os glóbulos brancos, os do sistema nervoso central e linfomas (sistema linfático). “Merece atenção o neuroblastoma, um tipo de câncer que ataca as células do sistema nervoso periférico, que se localizam no abdômen; o tumor de Wilms (renal); o retinoblastoma (ocular); o germinativo (das células que criam os ovários ou os testículos); o osteossarcoma (ossos); e os sarcomas (acometem as partes moles)”, enumera o oncopediatra.

A enfermidade representa a principal causa de morte (8% do total) entre crianças e adolescentes de 0 a 19 anos no Brasil, por isso, a importância da data para a conscientização do diagnóstico precoce. “Ao observar os sintomas, procurar o médico para diagnóstico e começar o tratamento. Na fase inicial da doença, as chances de cura, além de serem maiores, levam o paciente a uma melhor qualidade de vida em comparação com o diagnóstico em estágio avançado”, conclui Dr. Vicente Odone.

Sobre o ITACI – Instituto de Tratamento do Câncer Infantil
Construído a partir de uma parceria entre a Fundação Criança, Ação Solidária Contra o Câncer Infantil (ASCCI) e a Fundação Oncocentro de São Paulo, o ITACI é um hospital público que iniciou suas atividades em 2002 e oferece tratamento a crianças e adolescentes, portadores de câncer e outras doenças hematológicas ou raras, de 0 a 18 anos. Atuante na área de oncologia pediátrica no Brasil, realizou em seu ambulatório, em 2023, mais de 28 mil consultas médicas e multiprofissionais, cerca de 5 mil sessões de quimioterapia e 20 transplantes, entre eles, autólogo, alogênico aparentado e alogênico não aparentado. Entre as especialidades atendidas, leucemias, linfomas, neuroblastomas, sarcomas das partes moles, retinoblastomas, doenças hematológicas não oncológicas e tumores sólidos (do sistema nervoso central, renais, hepáticos, ósseos, e das células germinativas).


Dia Mundial do Câncer: é possível evitar e detectar o tumor de forma precoce? 


Alguns hábitos podem contribuir para minimizar as chances de uma pessoa receber um diagnóstico de câncer — Foto: Freepik (imagem ilustrativa)

Estimativa do Instituto Nacional do Câncer (Inca) é que, até 2025, o Brasil deverá registrar - por ano- 704 mil novos casos dos vários tipos da doença

Quando se fala em câncer é quase impossível não se esbarrar no que se refere ao diagnóstico precoce. Mas, aí surge a dúvida: é possível identificar a propensão de se desenvolver algum tipo de tumor bem antes de ele se manifestar ou potencializar? Essa preocupação se torna ainda mais relevante, quando se observa a estimativa do Instituto Nacional do Câncer (Inca) que, até 2025, o Brasil deverá registrar 704 mil novos casos de câncer por ano. 

Segundo o instituto, o tumor maligno mais incidente na população brasileira continuará sendo o de pele não melanoma. O levantamento destaca que o câncer de mama — o primeiro mais incidente nas mulheres — deverá provocar 74 mil novos casos anualmente no triênio 2023-2025. Já o de próstata — o primeiro mais recorrente nos homens — totalizará 72 mil diagnósticos. 

Diante dessas expectativas, e com o Dia Mundial de Combate ao Câncer — celebrado neste 4 de fevereiro — como pano de fundo, o oncologista clínico e coordenador do Hospital Integrado do Câncer Mater Dei, de Salvador (BA), Cleydson Santos, detalhou as formas de diagnóstico precoce e de prevenção que podem ajudar a população a se cuidar para não fazer parte das estatísticas. 

Conforme o especialista, os exames genéticos para rastreamento são interessantes, mas não são validados para usar no público em geral. “Eles são realizados somente em pacientes já diagnosticados com câncer, normalmente nos mais jovens, ou nas pessoas que tenham um histórico familiar de recorrentes tumores malignos”, afirma. 

Já em relação aos demais testes para investigar a presença do câncer, o médico destaca que eles são utilizados com o intuito de detectar a doença em seu estágio inicial. “A mamografia, por exemplo, é uma forma de identificar o tumor de mama; o exame de toque e retal de PSA, para o de próstata; a colonoscopia, para o de colorretal; o preventivo, para câncer de colo uterino, dentre outros.” 

Caminhos para prevenção
O oncologista chama atenção para uma preocupação muito comum entre as pessoas: se o câncer é hereditário. “Apenas 10% dos tumores são passados de pai para filhos. Os outros 90% são esporádicos, ou seja, desenvolveram-se por outros fatores. Outro ponto importante é que cerca de 50% dos casos poderiam ser evitados”, alerta Cleydson.

Sobre as formas de prevenção, o médico lista algumas práticas que contribuem para minimizar as chances de um diagnóstico de câncer maligno:

· abandono do tabagismo: o hábito de fumar cigarro está relacionado ao câncer de pulmão, esôfago, bexiga, dentre outros;

· consumo moderado de bebida alcoólica: quem ingere álcool com frequência aumenta o risco de desenvolver diferentes tipos de tumor como o de fígado, faringe, laringe, esôfago e outros;

· prática regular de atividade física: cerca de 50 minutos de exercícios aeróbicos por semana previne a obesidade, que está ligada a vários tumores, como de mama, intestino, etc;

· dosar a ingestão de alimentos industrializados: o consumo em excesso de enlatados e ultraprocessados, assim como de açúcar, aumentam o risco de câncer de intestino, de esôfago, estômago, dentre outros;

· uso de protetor solar: evitar tomar sol das 10h às 16h, reduz as chances de ser diagnosticado com câncer de pele (o mais comum no Brasil);

· utilização de preservativos nas relações sexuais: alguns vírus transmitidos durante o sexo desprotegido podem desencadear tumores. O HIV, por exemplo, pode causar leucemia e outros cânceres de linfomas. Já o HPV pode potencializar o de colo de útero;

· vacinação: a imunização contra a Hepatite B e o HPV reduz as possibilidades de câncer de fígado e de colo de útero, respectivamente.

A imunização aliada a ampliação do rastreamento na saúde básica são fundamentais para a prevenção e para o diagnóstico precoce do câncer, segundo o especialista. “As pessoas precisam entender o impacto disso e saber ainda que os números são subestimados. Mas apesar disso, e, ao contrário de antigamente, os tumores não são uma sentença de morte, porque a qualidade dos tratamentos e cirurgias melhoraram muito”, finaliza Cleydson. (Fonte: Por Nubya Oliveira)


Combinações de remédios e alimentos podem ser perigosos 



Na maioria das vezes, é recomendado comer algo antes de ingerir qualquer medicamento. Entretanto, é importante considerar que os alimentos têm um grande efeito na forma como o corpo processa os medicamentos, podendo aumentar ou reduzir a eficácia do tratamento.

Algumas combinações de medicamentos e alimentos podem resultar em coágulos sanguíneos ou danos no fígado. Segundo o doutor em Farmacologia e coordenador do curso de Farmácia da Universidade Positivo (UP), Felipe Lukacievicz Barbosa, é crucial revisar a bula antes de iniciar um novo tratamento para identificar alimentos que podem ou não ser consumidos durante o uso do remédio. 

Confira a seguir, cinco combinações que, segundo o especialista, devem ser evitadas.

· Antibióticos x laticínios
A ingestão de alguns medicamentos, como certas classes de antibióticos (tetraciclinas e quinolonas), pode ter um efeito negativo se feita com leite. O cálcio presente no leite ou em derivados pode formar um complexo insolúvel com as moléculas dos antibióticos, impedindo a absorção normal pelo corpo. “Ou seja, o organismo pode não absorver a quantidade correta do medicamento, comprometendo o efeito farmacológico. Logo, a doença pode não ser tratada efetivamente”, explica. Mas isso não significa que nenhum medicamento pode ser tomado com leite.  Por isso, na dúvida, é sempre importante consultar um médico ou farmacêutico.

· Estatinas x frutas cítricas
Algumas frutas cítricas, como o suco de toranja (grapefruit), podem conter moléculas que interferem na absorção das Estatinas, medicamentos que exercem o efeito de diminuir os níveis de colesterol no sangue. “Quando ingeridas com esses sucos, as estatinas sofrem um aumento elevado, sendo interpretado como um efeito tóxico no sangue. Apesar de os dados ainda não relacionarem diretamente essa associação de maneira perigosa, na dúvida, é melhor ingerir os medicamentos com água”, explica.

· Anticoagulantes x hortaliças folhosas
As hortaliças folhosas, como espinafre, couve e alface, apresentam abundantes quantidades de Vitamina K, que é  crucial na ativação da cascata da coagulação sanguínea, quando necessário. No entanto, para aqueles que fazem uso de anticoagulantes, cujo objetivo é inibir essa cascata por diversos motivos, a ingestão excessiva dessas hortaliças podem reduzir a eficácia do medicamento, ocasionando a ativação indesejada da cascata de coagulação. Fazendo com que o medicamento perca seu efeito e a pessoa volte a formar coágulos no sangue.

· Analgésicos x bebidas alcoólicas
O consumo de bebidas alcoólicas não deve ser evitado apenas ao tomar antibióticos. Antidepressivos, analgésicos e medicamentos para diabete também podem causar uma série de complicações quando interagem com o álcool. A combinação de analgésicos, como ácido acetilsalicílico e paracetamol, com bebidas alcoólicas também apresenta riscos significativos. Além do aumento do risco de sangramento gástrico, a mistura pode levar à toxicidade hepática, respectivamente. O efeito depressor do álcool no sistema nervoso central também pode potencializar os efeitos dos analgésicos, resultando em sedação excessiva e outros sintomas indesejados.

· Antipsicóticos x café
Os antipsicóticos, projetados para diminuir a atividade do sistema nervoso central, podem ter o efeito comprometido pelo consumo de cafeína presente no café. Em casos de tratamento para condições como esquizofrenia, em que a sedação é desejada, o café pode contrariar esse efeito, causando agitação, distúrbios do sono e aumento da frequência cardíaca. O equilíbrio entre a medicação e o consumo de cafeína deve ser cuidadosamente avaliado pelo profissional de saúde.

“Conhecer e compreender essas interações é essencial para garantir a eficácia dos tratamentos e evitar potenciais complicações à saúde. Sempre consulte um médico ou farmacêutico antes de fazer alterações significativas em sua dieta, ou rotina de medicamentos”, aconselha Barbosa.

Sobre a Universidade Positivo
A Universidade Positivo é referência em Ensino Superior entre as IES do Estado do Paraná e é uma marca de reconhecimento nacional. Com salas de aula modernas, laboratórios com tecnologia de ponta e mais de 400 mil metros quadrados de área verde no campus sede, a Universidade Positivo é reconhecida pela experiência educacional de mais de três décadas. A Instituição conta com três unidades em Curitiba (PR) e uma em Londrina (PR), e mais de 70 polos de EAD no Brasil. Atualmente, oferece mais de 60 cursos de graduação, centenas de programas de especialização e MBA, cinco programas de mestrado e doutorado, além de cursos de educação continuada, programas de extensão e parcerias internacionais para intercâmbios, cursos e visitas. Além disso, tem sete clínicas de atendimento gratuito à comunidade, que totalizam cerca de 3.500 metros quadrados. Em 2019, a Universidade Positivo foi classificada entre as 100 instituições mais bem colocadas no ranking mundial de sustentabilidade da UI GreenMetric. Desde março de 2020 integra o Grupo Cruzeiro do Sul Educacional. Mais informações em up.edu.br/


'Alergia - O Corpo em Alerta': entenda como as alergias se desenvolvem e por que algumas pessoas são mais sensíveis 

Alergia ataca sistema de defesa do corpo humano. Foto: TV Globo/Reprodução

Momento do parto é fundamental para o desenvolvimento de alergias. Foto: TV Globo/Reprodução

Na nova série do Fantástico, o doutor Drauzio Varella vai conversar com especialistas e acompanhar a rotina de pessoas que sofrem com as restrições alérgicas

Se você não tem, certamente conhece alguém que tenha. A estimativa é de que 61 milhões de brasileiros sofram com algum tipo de alergia. Respiratória, de pele, alimentar, medicamentosa... são muitas as opções.

Mas, afinal, por que uns têm e outros não? O Doutor Drauzio Varella conversou com especialistas e acompanhou a rotina de pessoas que têm uma vida repleta de restrições - e riscos - por causa da doença. O que acontece dentro do corpo? Como as alergias se desenvolvem?

Quais são os tratamentos promissores? Esses tópicos são discutido na estreia da série "Alergia - O Corpo em Alerta".

O que acontece dentro do corpo?
A alergia é a reação exagerada e equivocada do nosso sistema imunológico contra alguma substância inofensiva.

"O sistema imune está em tudo, está na pele, está nos olhos, está na mucosa respiratória, está no aparelho digestivo. Uma vez que esse radar detectou algo estranho, ele pode amplificar e ligar toda essa rede de comunicação desde uma reação localizada até uma reação que acometa o corpo todo", destaca Ariana Campos Yang, Médica alergista do HC-USP.

A pessoa vai então espirrar, tossir, os olhos vão lacrimejar. Pode também ter inchaços, vômitos, diarreia, coceiras. Na maioria dos casos, essas reações são leves - o próprio organismo resolve.

A intensidade desses sintomas vai depender da quantidade de histamina liberada nesse processo. A histamina é um mediador químico - um mensageiro que controla as reações alérgicas.

"Quando uma reação alérgica acontece, é como se tivesse chovendo histamina no corpo, tivesse chovendo essa substância. Só que, às vezes, a gente tem uma chuva, uma garoinha E, às vezes, a gente tem uma tempestade, um vendaval que causa uma enchente, destruição", completa a médica alergista.

Por que uns têm alergia e outros não?
"É um grande mistério saber por que uma pessoa se torna alérgica. O que nós sabemos é que a alergia é resultado de uma conversa que envolve predisposição genética com fatores ambientais, como poluição, alimentação e medicações (...) o nosso sistema imune começa a ser formado já desde quando a gente está na barriga da nossa mãe, o que tem a ver com a alimentação da mãe e com o estilo de vida dela enquanto o bebê está sendo gerado".

Segundo a especialista, outro momento crucial é o parto, onde há uma força ambiental para predispor à alergia.

"As bactérias presentes no canal vaginal sintonizam um perfil mais regulador, evitando fatores que predisponham à alergia. Quando o bebê nasce por cesariana, as bactérias na pele do bebê não oferecem a mesma ajuda. Isso não significa que quem nasce por cesárea terá ou não alergia, mas é uma variável que influencia".

Outra coisa importante a ser destacado é que a maioria das alergias é mais frequente na infância porque o sistema imune ainda está em desenvolvimento.

Como o Brasil trata as pessoas com alergia?
"A rede pública não está preparada para a questão da alergia. Existem doenças desconhecidas, e é necessário formar mais alergistas. A distribuição de alergistas no Brasil é muito desigual, com cerca de 70% concentrados na região sudeste", destaca Fábio Kuschnir, Presidente da Associação Brasileira de Alergia e Imunologia.

Felipe Proenço, Diretor de Programa da Secretaria de Atenção Primária à Saúde, diz que a atenção primária é fundamental para combater problema.

"Primeiro, é crucial garantir a presença da atenção primária em todas as localidades. No início de 2023, 5.000 equipes de saúde da família estavam sem médico. Além disso, o teleatendimento é uma estratégia importante para conectar médicos especialistas a regiões onde a presença física não é viável", afirma.

Tratamentos promissores
A reportagem acompanhou em São Paulo, o dia a dia dos dois maiores centros brasileiros de tratamento de alergias. Há alguns tratamentos que estão melhorando a qualidade de vida dos pacientes.

"O grande avanço na área de alergia é entender que essas doenças têm um alvo bem específico que alimenta e organiza toda essa inflamação. Temos as novas terapias, conhecidas como terapias-alvo, que são injetáveis e agem como mísseis teleguiados, indo diretamente para o ponto específico", diz a médica alergista do HC-USP.

Crianças amamentadas por mais tempo tem QI mais alto, especialista explica 

Apesar do papel da genética na inteligência, outros fatores, como o tempo de amamentação também influenciam nesse desenvolvimento, afirma o Pós PhD em neurociências e especialista em genômica, Dr. Fabiano de Abreu Agrela

A inteligência é influenciada por diversos fatores, desde genéticos a ambientais, que podem afetar, seja positivamente ou negativamente, essa habilidade desde muito cedo. Por isso, cada vez mais estudos têm se dedicado a entender melhor como funciona esse processo de desenvolvimento e as melhores escolhas para ajudá-lo a ocorrer da melhor forma possível.

Um dos fatores que tem recebido destaque nos últimos tempos é a genética, mas apesar de ser um dos mais expressivos, existem outros pontos que podem alterar o desenvolvimento, como a amamentação, como afirma o Pós PhD em neurociências e especialista em genômica, Dr. Fabiano de Abreu Agrela.

“O desenvolvimento intelectual das crianças é influenciado tanto pela herança genética quanto pelas experiências ambientais. Gosto de fazer uma analogia de uma corrida entre dois carrinhos com rodas, de mesmo peso e mesma reta, a serem empurrados. O carrinho empurrado com mais força chegará mais rapidamente em seu objetivo. A amamentação está associada ao capital humano, que está associado ao QI.”

Amamentação e QI: Uma relação próxima?
De acordo com o Dr. Fabiano de Abreu Agrela, o período de amamentação ajuda no desenvolvimento da inteligência da criança.

“As crianças amamentadas atingem pontuações de QI mais altas do que as crianças não alimentadas com leite materno, presumivelmente por causa dos ácidos graxos exclusivamente disponíveis no leite materno, mas também pelos nutrientes essenciais obtidos no aleitamento.”

Existem estudos científicos que ajudam a reforçar a relação entre inteligência e o período de amamentação, como o ‘Moderação dos efeitos da amamentação no QI por variação genética no metabolismo dos ácidos graxos’, que demonstrou que a associação entre amamentação e QI é moderada por uma variante genética no FADS2, um gene envolvido no controle genético das vias dos ácidos graxos. Catalogada no GWAS. 

“O estudo, que foi publicado no Journal of Pediatrics, mostrou que a fórmula suplementada com membrana de glóbulos de gordura do leite (MFGM) e lactoferrina por um ano aumentou o QI das crianças em 5 pontos aos 5 ½ de idade. O efeito foi mais perceptível na velocidade de processamento de informações e nas habilidades visual-espacial das crianças.”

Qual é o período ideal de amamentação para o bebê?
O período de amamentação do bebê pode variar de caso a caso, mas em geral, os estudos baseiam-se em uma média de 12 meses de aleitamento materno, sendo que os 6 primeiros meses devem contar com o leite materno de forma exclusiva.

“A amamentação e a inteligência têm uma forte relação, mas essa informação não deve ser tida como uma regra. Temos variantes sentinelas em relatório genético de inteligência que estão relacionadas à amamentação e que podem aumentar, somadas, até 10 pontos de QI. Mas há muitas outras variantes relacionadas ao QI e não somente as relacionadas à amamentação”, ressalta Dr. Fabiano.

Sobre Dr. Fabiano de Abreu Agrela
Dr. Fabiano de Abreu Agrela é Pós PhD em Neurociências e biólogo membro das principais sociedades científicas como SFN - Society for Neuroscience nos Estados Unidos, Sigma XI, sociedade científica onde os membros precisam ser convidados e que conta com mais de 200 prémios Nobel e a RSB - Royal Society of Biology, maior sociedade de biologia sediada no Reuno Unido. É membro de 10 sociedades de alto QI, entre elas a Mensa, Intertel, ISPE, Triple Nine Society, coordenador Intertel Brazil, diretor internacional da IIS Society e presidente da ISI e ePiq society, todas sociedades restritas para pessoas com alto QI comprovados em testes supervisionados. Criou o primeiro relatório genético que estima a pontuação de QI através de teste de DNA e o projeto GIP - Genetic Intelligence Project com estudos genéticos e psicológicos sobre alto QI com voluntários. Autor de mais de 50 estudos sobre inteligência, foi voluntário em testes de QI supervisionados, testes genéticos de inteligência e estudo de neuroimagem já que atingiu a pontuação máxima em mais de um teste de QI em mais de um país corroborando com os demais resultados genéticos e de neuroimagem.

Hanseníase persiste como desafio global de saúde, revelam dados recentes 


Doença crônica e contagiosa que afeta a pele e as células que dão suporte e proteção aos neurônios. O Brasil é o segundo país com maior incidência de casos

A hanseníase, uma das enfermidades mais antigas da humanidade, continua a representar um desafio significativo para a saúde pública em diversas regiões do mundo, incluindo o Brasil. Apesar dos avanços no diagnóstico e tratamento ao longo das últimas décadas, a doença ainda mantém uma presença marcante em nível global.

De acordo com informações divulgadas, em 2018, a hanseníase permanecia uma preocupação para 22 países, com altas incidências em todo o mundo. A Índia liderava os casos, seguida pelo Brasil em segundo lugar e a Indonésia completando o cenário. No Brasil, entre 2016 e 2020, foram registrados 155.359 novos casos, com maior prevalência entre homens (55,5%) e pessoas pardas (58,9%), principalmente nas regiões Sul e Sudeste. A análise demonstrou que a incidência está correlacionada inversamente com os níveis educacionais, com 40,9% dos casos entre indivíduos com ensino fundamental incompleto.

Os dados de 2020 do Ministério da Saúde revelaram que o Brasil diagnosticou 17.979 novos casos, representando 93,6% do total nas Américas. Essa estatística mantém o país classificado pela OMS como um local de alta incidência da doença, ocupando o segundo lugar no ranking global, atrás somente da Índia. Apesar de uma leve diminuição nos registros nos últimos anos, o Ministério da Saúde suspeita que a redução possa ser resultado da subnotificação durante a pandemia de COVID-19.

De acordo com a dermatologista Simone Neri, a hanseníase é uma enfermidade crônica e contagiosa causada pelo Mycobacterium lepra, que afeta a pele e as células que dão suporte e proteção aos neurônios e sistema nervoso periférico. “A transmissão ocorre predominantemente pelas vias respiratórias com consequências que incluem incapacidades físicas e deformidades que afetam a vida social e psicológica dos pacientes, gerando estigma e preconceito. Essa estigmatização tem raízes históricas antigas, não se limitando apenas às dificuldades físicas”, explica a Dra. Simone Neri.

Campanha Janeiro Roxo:
A luta contra a hanseníase ainda é um desafio, por este motivo existe o Janeiro Roxo, Campanha do Ministério da Saúde voltada para conscientização e combate à hanseníase. Durante o mês de Janeiro, diversas atividades são realizadas para disseminar informações sobre a hanseníase, seus sintomas, formas de prevenção, diagnóstico precoce e tratamento adequado.

O objetivo é reduzir o estigma em torno da doença, promover o acesso aos serviços de saúde para diagnóstico e tratamento, além de incentivar a busca por ajuda médica ao menor sinal de sintomas relacionados à hanseníase.

“A campanha busca não só informar a população em geral, mas também sensibilizar profissionais de saúde, educadores e autoridades para que haja um diagnóstico mais rápido e eficaz, contribuindo para a redução da incidência e para a melhoria na qualidade de vida das pessoas afetadas por essa condição”, diz a Dra. Simone Neri.

Tratamento:
Segundo a Dra. Simone Neri, o tratamento da hanseníase geralmente envolve o uso de antibióticos específicos para combater a infecção causada pela bactéria Mycobacterium leprae. A duração e o tipo de tratamento podem variar de acordo com a forma clínica da doença, sendo classificada em hanseníase paucibacilar (PB) e multibacilar (MB).

“A Hanseníase Paucibacilar (PB) é o tipo menos grave e pode ser tratado com uma combinação de medicamentos, que são administrados durante seis meses a um ano. Já a Hanseníase Multibacilar (MB) é um tipo mais grave e seu tratamento precisa de mais medicamentos e um período de um a dois anos. Esse tratamento é oferecido gratuitamente pelos serviços de saúde e é crucial segui-lo rigorosamente para garantir a eliminação da bactéria, prevenir resistência aos medicamentos e reduzir o risco de complicações. Além da terapia medicamentosa, também é fundamental monitorar regularmente a pessoa em tratamento para avaliar a melhora clínica, identificar possíveis efeitos colaterais dos medicamentos e oferecer apoio para lidar com possíveis consequências físicas, emocionais e sociais da doença”, finaliza a Dra. Simone Neri.

- Dra. Simone Neri –/ DERMATOLOGISTA E TRICOLOGISTA/ - CRM 80.919/ - Possui 30 anos de formação em Clínica Médica e em Dermatologia. É graduada em Medicina pela Universidade de Santo Amaro UNISA, possui residência em Clínica Médica pela Universidade de Santo Amaro UNISA, residência em Dermatologia pela Universidade de Santo Amaro UNISA. Foi preceptora do Ambulatório de Tricologia da Faculdade de Medicina da Universidade de Santo Amaro UNISA, médica plantonista do Pronto Socorro do Hospital São Luiz e coordenadora médica do Pronto Socorro do Hospital São Luiz Anália Franco./ Na área de inovações em técnicas cirúrgicas, participou de um grupo de estudos no Instituto Butantã no tema Toxina Botulínica em Processos Inflamatórios do Couro Cabeludo, com apresentações em Congressos Nacionais e Internacionais. Atualmente, realiza Pós-Graduação em Cosmiatria com foco em Tecnologias como Lasers, Ultrassom Microfocado/ e Radiofrequência, além da consagrada técnica de Harmonização Facial “Matemática da Beleza”. Já na área da Dermatologia Clínica investe exaustivamente em atualizações científicas, com tratamentos inovadores como os chamados Medicamentos Imunobiológicos em doenças crônicas como Psoríase e Hidrosadenite. Na área de gestão é Diretora do grupo Simone Neri (Clínica Simone Neri, Academia Skinnews, Skinnews Estética), em Osasco-SP.

Cuidados com a saúde das crianças 


Doenças como desidratação, intoxicação alimentar, insolação, otites e acidentes domésticos são mais comuns nessa época de férias e temperaturas elevadas; Saiba como prevenir e proteger crianças e bebês

O Verão começa oficialmente no próximo dia 22 de dezembro, mas, as altas temperaturas já chegaram a diversas regiões do país, favorecendo doenças típicas dessa época, como desidratação, insolação, intoxicações alimentares e doenças de pele. 

“Já estamos enfrentando ondas de calor significativas, que trazem riscos à saúde, principalmente das crianças que são mais vulneráveis, já que o sistema imunológico ainda está em desenvolvimento”, ressalta Dra. Fabíola La Torre, Coordenadora Médica da Linha Pediátrica do Hospital São Luiz Osasco. 

Além das doenças, no período de férias, onde as crianças têm mais tempo livre, também é importante ficar atento aos acidentes domésticos, como afogamentos, quedas e queimaduras. 

“E mesmo no verão, é preciso se preocupar com gripes, resfriados e pneumonia, que podem ocorrer em qualquer época do ano”, lembra a pediatra do São Luiz Osasco.

A unidade da Rede D’Or, que conta a maior e mais completa estrutura hospitalar da cidade de Osasco e uma linha de cuidado pediátrico de referência, elaborou uma lista de cuidados e orientações de prevenção para ajudar nos cuidados com as crianças, que integra todos os receituários de atendimentos realizados no pronto-socorro infantil.
 
Confira algumas dicas: 
 - Use protetor solar. 
Utilize produtos específicos para o público infantil, com fator de proteção alto e quantidade adequada. Não se esqueça de regiões sensíveis como pés, orelhas e nuca. Reaplique sempre que a criança sair da água e várias vezes ao longo do dia. 

Atenção, bebês com menos de seis meses não devem ser expostos diretamente ao sol ou utilizar filtro solar. Neste caso, é indicado o uso de barreiras mecânicas, como roupas e guarda-sol, que precisa ser de tecido escuro, como lona.
 
- Atenção com a exposição ao sol. 
Evite os horários de maior incidência dos raios solares, entre 10h e 16h. Além do protetor solar, use chapéus e roupas com proteção UV.

Lembre-se, o efeito da radiação solar é acumulativo e o principal fator de risco para o câncer de pele.
 
- Mantenha as crianças hidratadas. 
Ofereça água e outros líquidos naturais, como sucos e água de coco frequentemente. 

“Não espere a criança pedir, pois ela tende a fazer isso quando a situação já está grave. É importante ficar atento ao suor, urina e diarreia. Caso identifique sintomas como boca seca, pouca urina, sonolência ou pele acinzentada inicie o processo de hidratação e busque atendimento médico”, orienta a pediatra.
 
- Cuidado com a alimentação. 
Com o calor, as comidas estragam rapidamente. Por isso, atenção à conservação e refrigeração dos alimentos e evite consumir itens prontos fora de casa e principalmente nas praias. Alimentos como ovo, carnes, peixes e maionese são os mais suscetíveis. Opte sempre por opções mais leves, naturais e saudáveis. 

“Outra tendência das férias é liberar o consumo de guloseimas. Fique atento à quantidade e não deixe isso virar rotina”, alerta Dra. Fabíola.
 
- Atenção com a pele.
O contato constante com a água, seja pela transpiração ou mergulho em praias e piscinas, faz com que a pele fique úmida por mais tempo, favorecendo o aparecimento de micoses, doença causada por fungos. Ao identificar lesões vermelhas, escamação da pele e coceira, procure um dermatologista.

“Para prevenir, o ideal é secar bem o corpo, principalmente o meio dos dedos dos pés. Além disso, para evitar outras alergias, sempre lave o corpo com água limpa, para retirar cloro, areia e outras substâncias”, reforça a coordenadora do Hospital São Luiz Osasco.
 
- Cuidado com os ouvidos. 
O excesso de água elimina a cera na parte interna do ouvido, reduzindo a proteção e favorecendo quadros de otite. A orientação é secar a água do ouvido, inclinando a cabeça da criança para os dois lados, e usar protetores.
 
- Modere o uso do ar-condicionado.
O equipamento está liberado e pode ser um ótimo aliado para amenizar as temperaturas, mas com alguns cuidados. O aparelho não pode ser instalado próximo ao berço ou cama dos bebês e a temperatura deve ser amena, em torno de 25 graus. Além disso, limpe os filtros semanalmente e busque umidificar o ambiente.
 
- Evite acidentes.
 Em locais com piscina, é essencial incluir barreiras físicas, como grades e portões. No caso de bebês, apenas 2,5 centímetros de água são suficientes para ocasionar afogamento, por isso, atenção também com baldes, bacias, banheiros e lavanderias.

Cuidado com cordões, fios e cordas, que podem causar estrangulamento, assim como itens inflamáveis e tóxicos, como produtos de limpeza, além de tomadas e itens elétricos. Não deixe panelas quentes no forno ou fogão e sempre mantenha os cabos voltados para o lado interno. 

“São diversos riscos presentes em elementos comuns do nosso cotidiano, por isso, nunca deixe as crianças sem supervisão, mesmo em casa”, reforça Dra. Fabíola.

Otoplastia: conheça a cirurgia das orelhas 


A otoplastia é uma cirurgia plástica que tem como objetivo corrigir a forma, a posição e/ou o tamanho das orelhas. Embora muitas vezes seja considerada uma intervenção estética, a otoplastia vai além da aparência física, pois também pode ter um impacto significativo na autoestima e na confiança do paciente.

Por meio dessa cirurgia, é possível corrigir deformidades ou alterações nas orelhas, o que pode resultar em uma melhora na qualidade de vida do indivíduo.

Pensando nesse procedimento, hoje vamos conversar sobre:

· O que é otoplastia?

· Como é realizado o procedimento?

· Quais são as expectativas da otoplastia?

· Cuidados pós-operatórios.

O que é otoplastia?
A autoestima é uma parte essencial da saúde mental e emocional de uma pessoa. Muitas vezes, características físicas que estão fora dos padrões sociais podem causar desconforto e insegurança.

Por meio da cirurgia plástica, é possível corrigir essas características e ajudar o paciente a se sentir mais confiante e satisfeito com sua aparência.

No caso da otoplastia, a cirurgia visa principalmente corrigir orelhas proeminentes, também conhecidas como “orelhas de abano”.

Essa condição ocorre quando as orelhas se projetam para fora em um ângulo mais aberto do que o “normal”, causando um aspecto desproporcional em relação ao rosto.

Além disso, a otoplastia também pode corrigir deformidades congênitas, assimetrias e outros problemas estéticos relacionados às orelhas.

Como é feita a cirurgia?
O primeiro passo antes da realização da otoplastia é uma consulta com um cirurgião plástico especializado.

Durante essa consulta, o médico avaliará a condição das orelhas, discutirá sobre as expectativas do paciente e esclarecerá todas as dúvidas sobre o procedimento.

É importante que o paciente esteja em boas condições de saúde geral e tenha expectativas realistas em relação aos resultados da cirurgia.

No dia da cirurgia, o paciente geralmente recebe anestesia local com sedação ou anestesia geral, dependendo das preferências do cirurgião e do paciente. A otoplastia pode ser realizada em uma clínica ou em um hospital, dependendo da complexidade do caso.

O procedimento em si começa com o cirurgião fazendo incisões na parte de trás das orelhas. Essas incisões são posicionadas de forma estratégica para que as cicatrizes delas resultantes fiquem bem escondidas.

Em seguida, o cirurgião acessa a cartilagem das orelhas e, se necessário, remove ou remodela parte dela para alcançar o resultado desejado, de forma que fiquem mais próximas da cabeça.

Suturas internas mantêm a nova forma da orelha e o cirurgião também pode remover o excesso de pele, se necessário, para obter um resultado mais harmonioso.

O tempo necessário para a realização da otoplastia varia de acordo com a complexidade do caso, mas, geralmente, leva de uma a duas horas.

Quais são expectativas da otoplastia?
É importante que o paciente tenha expectativas realistas em relação aos resultados após a realização da cirurgia de otoplastia. 

Embora a otoplastia seja eficaz na correção de deformidades ou de alterações estéticas das orelhas, é essencial entender que o resultado final pode variar de acordo com a anatomia individual, a habilidade do cirurgião e o processo de cicatrização.

A principal expectativa que o paciente pode ter é a correção das características indesejadas das orelhas.

No caso das orelhas proeminentes, a otoplastia busca reposicioná-las de forma mais próxima da cabeça, proporcionando um perfil mais equilibrado e natural. Com a correção da deformidade, é possível obter uma aparência mais harmoniosa e proporcional em relação ao rosto.

Apesar das melhorias estéticas proporcionadas pela otoplastia, é importante ter em mente que a cirurgia não transformará completamente a aparência das orelhas ou do rosto.

É fundamental compreender que a otoplastia é um procedimento para aprimorar a estética das orelhas, mas não irá alterar drasticamente a estrutura facial ou outras características físicas.

Cuidados pós-operatórios
Após a conclusão da cirurgia, o paciente é levado para uma sala de recuperação, onde será monitorado até que esteja estável e acordado.

São normais sentir algum desconforto e surgirem inchaço e hematoma ao redor das orelhas nos primeiros dias após a cirurgia. O cirurgião pode prescrever analgésicos e recomendar compressas frias para ajudar a aliviar esses sintomas.

É fundamental seguir todas as instruções do médico com relação aos cuidados pós-operatórios.

Isso pode incluir utilizar uma faixa compressiva para ajudar a manter as orelhas no lugar durante a cicatrização, evitar atividades físicas intensas e dormir de lado durante as primeiras semanas após a cirurgia. 

O paciente também deve evitar expor as orelhas a traumas ou fortes impactos durante o período de recuperação.

É importante ressaltar que os resultados da otoplastia não são imediatamente visíveis, uma vez que o inchaço precisa diminuir e as orelhas precisam se adaptar à nova forma.

Fonte: Dr. Alexandre Kataoka, Cirurgião Plástico. Perito concursado da Secretaria da Justiça de São Paulo - Instituto de Medicina Social e Criminologia do Estado de São Paulo. Diretor Adjunto do DEPRO - órgão fiscalizador da Sociedade Brasileira de Cirurgia Plástica (SBCP). Consultor Jurídico da SBCP-SP. Representante da SBCP no CODAME/CFM. Membro Efetivo da Câmara Técnica em cirurgia plástica - CFM. Coordenador da Câmara Técnica do CREMESP.

Herpes ocular: apesar de rara, doença pode levar à cegueira se não for tratada a tempo, alerta especialista do H.Olhos 

O vírus pode ficar incubado no organismo desde a primeira infância, sem que a pessoa nunca saiba quando foi contaminada

Se engana quem pensa que o herpes respeita barreiras, ficando restrito aos lábios ou à região genital. Causada pelo vírus do herpes simples (Herpes Simplex Virus, HSV), a doença também pode se desenvolver em outras partes ainda mais sensíveis do corpo, como os olhos. Ao serem infectados, pode ocorrer necrose aguda da retina - quando há um processo infeccioso ou inflamatório que destrói e causa morte do tecido retiniano (aquela parte do olho que transforma luz em estímulo nervoso e o envia ao cérebro), levando a pessoa à cegueira.

Especialista em córnea do H.Olhos - Hospital de Olhos e Professora da Escola Paulista de Medicina da Universidade Federal de São Paulo (Unifesp), Dra. Denise Freitas explica que, em geral, o herpes é adquirido na infância, por contato direto com vírus. “Por exemplo, alguém que esteja com herpes labial ativo e beija outra pessoa, acaba se infectando. O vírus pode não se manifestar clinicamente, mas fica latente (disfarçado) no corpo, por longo período. Durante esse tempo, e por motivos variados, que incluem alteração da resposta imunológica de proteção, o vírus latente pode iniciar sua multiplicação, causando a doença que é considerada uma recorrência e, nos olhos, o contágio pode aparecer nas pálpebras que, por aproximação, acaba afetando o olho”.
 
Doença oportunista
Segundo a médica, a contaminação pelo herpes independe de onde ele está localizado no corpo, mas como o olho está na região do rosto, o herpes facial/labial acaba sendo o mais frequentemente relacionado no contágio. Além disso, embora ninguém esteja imune ao vírus, algumas pessoas estão mais predispostas à doença, como os pacientes atópicos (alérgicos), principalmente os mais severos e, também, os pacientes imunodeprimidos. “A identificação da doença é feita pelo exame clínico oftalmológico. Excepcionalmente, são necessários exames para confirmar o diagnóstico”, diz a especialista.

É preciso se atentar aos sintomas da doença, pois nem sempre o herpes simples tem manifestação clínica (quando aparecem os primeiros sinais), podendo estar presente no organismo desde a primeira infância, sem que a pessoa nunca saiba quando foi contaminada pelo vírus. Os primeiros sintomas podem aparecer na forma de vesículas na pele - aquelas pequenas bolhinhas com líquido em seu interior, em geral, causando muita coceira local. 

No olho, as vesículas podem acometer as pálpebras e por contiguidade (aproximação) a córnea. “A infecção ativa na córnea causa a perda da sua transparência, sendo algumas vezes necessário realizar um transplante de córnea, já que a infecção do herpes na retina causa sua necrose com consequente perda da visão”, comenta a Dra. Denise.
 
Tratamento possível
Antes de explicar como é feito o tratamento, a Dra. Denise ressalta que a doença herpética (herpes) é tratável, embora não seja curável. Isso significa que, uma vez que se contrai o herpes, ele ficará para sempre no organismo. A dúvida é se o vírus vai ou não causar infecção, e se ela será recorrente. 

“Pelo fato do olho perder sua transparência, nos casos de córnea, ou cicatrizar irreversivelmente, nos casos de retina, quando há a contaminação pelo herpes nestes tecidos, o tratamento deve ser agressivo e iniciado o mais rápido possível. Quanto mais tempo o vírus fica no tecido, maior a perda da visão”, alerta Dra. Denise.

A córnea pode ser inicialmente tratada com remédio antiviral tópico, aplicado diretamente no olho, na forma de colírios e/ou pomadas. Já nos casos mais graves e agressivos, o tratamento deve ocorrer com antiviral oral. Porém, se houver acometimento da retina, a intervenção deve ser sempre sistêmica, com medicação oral ou até mesmo endovenosa, com internação do paciente, dependendo da gravidade do acometimento.


Tuberculose: diagnóstico precoce favorece tratamento e cura 

Em 2022 foram registrados 7,5 milhões novos casos da doença no mundo, segundo levantamento divulgado este mês pela OMS

Descoberta no século XIX, a tuberculose pode ser prevenida, tem cura, mas no ano passado foi a segunda doença infecciosa que mais matou em todo o mundo, depois da covid-19. Ela é também a principal causa de morte de pessoas com HIV e uma das principais relacionadas à resistência antimicrobiana. Em 2022, estima-se que foram 1,3 milhões de mortes por tuberculose, incluindo 167 mil pessoas com HIV no mundo.

Ainda que tenha estatísticas elevadas, existe vacina contra a tuberculose, disponível no Brasil desde a década de 1970, e tratamento eficaz disponível no SUS. 

Causada pelo bacilo de Koch, uma bactéria chamada Mycobacterium tuberculosis, a tuberculose é uma doença infecciosa e transmissível que afeta prioritariamente os pulmões, mas também pode acometer outros órgãos, podendo se tornar fatal se não tratada adequadamente. A transmissão acontece por via respiratória, por meio da tosse, fala ou espirros de uma pessoa infectada. O bacilo é sensível à luz solar e a circulação de ar ajuda na dispersão de partículas, por isso é importante manter ambientes ventilados e com luz natural para diminuir o risco de transmissão.

Entre os sintomas mais comuns da tuberculose estão a tosse seca ou com catarro por três ou mais semanas, febre vespertina, sudorese noturna e emagrecimento. Ao observar esses sintomas é fundamental procurar um médico.

“O diagnóstico é rápido e muito assertivo, feito por exames como a baciloscopia e, exames de biologia molecular, como PCR e também de cultura, todos realizados com amostra de escarro. Além de exames de imagem, como radiografia e tomografia de tórax”, explica o médico infectologista do Sabin Diagnóstico e Saúde, Alexandre Cunha.

O tratamento deve ser iniciado imediatamente após o diagnóstico positivo, por meio de medicamentos. “Apesar de ser uma doença descoberta há séculos, com uma vacina segura e eficaz, diagnóstico rápido e cura, a tuberculose ainda afeta milhares de pessoas mundo afora e segue ocasionando óbitos”, alerta Alexandre.

“A vacina BCG (bacilo Calmette-Guérin) é a principal medida preventiva, o imunizante protege contra formas mais graves da doença e deve ser aplicado ainda na infância, entre o nascer e até completar 5 anos de idade”, complementa o médico. O imunizante está disponível na rede pública e também no Sabin, onde pode ser adquirido pelo e-commerce, nas cidades onde há unidade de vacina da empresa.

Nos últimos anos, houve uma redução importante da cobertura vacinal da BCG. Até 2018, o índice de vacinação estava acima do patamar de 95%. Porém, desde 2019, a cobertura não ultrapassa os 88%. Essa queda representa aumento do risco de casos graves e óbito pela doença.

Anualmente, milhões de pessoas sofrem com a enfermidade mundialmente e mais de um milhão vão a óbito por ela, só o Brasil notifica a cada ano cerca de 4,5 mil mortes em decorrência da tuberculose, segundo o Ministério da Saúde.

Grupo Sabin | Referência em saúde, destaque em gestão de pessoas e liderança feminina, dedicado às melhores práticas sustentáveis e atuante nas comunidades onde está presente, o Grupo Sabin nasceu na capital federal, fruto da coragem e determinação de duas empreendedoras, Janete Vaz e Sandra Soares Costa, em 1984. Hoje conta com cerca de 7.000 colaboradores unidos pelo propósito de inspirar pessoas a cuidar de pessoas.

Presente em 15 estados, além do Distrito Federal, a empresa oferece serviços de saúde com excelência, inovação e responsabilidade socioambiental às 78 cidades em que está presente e atende mais de 6,5 milhões de clientes ao ano em 350 unidades distribuídas de norte a sul do país.

O ecossistema de saúde do Grupo Sabin integra um portfólio de negócios que contempla análises clínicas, diagnósticos por imagem, anatomia patológica, genômica, imunização e check-up executivo. Além disso, contempla também serviços de atenção primária contribuindo para a gestão de saúde de grupos populacionais por meio de programas e linhas de cuidados coordenados, e a plataforma integradora de serviços de saúde - Rita Saúde - solução digital que conta com diversos parceiros como farmácias, médicos e outros profissionais, promovendo acesso à saúde com qualidade e eficiência.

Oncologista explica melhor método para diagnosticar o câncer de próstata 

Seis em cada dez casos são identificados em homens com mais de 65 anos

De acordo com dados do Instituto Nacional de Câncer (INCA), o câncer de próstata é o segundo tipo mais frequente em homens brasileiros, atrás apenas do câncer de pele não melanoma, sendo também a segunda maior causa de morte, depois do de pulmão. Para o triênio 2023-2025, estimam-se 71.730 novos casos de câncer de próstata por ano.

Ao encarar esses dados, o diagnóstico precoce se torna uma opção crucial, pois não apenas aumenta significativamente as perspectivas de cura, mas também reduz a taxa de mortalidade, evita o sofrimento e minimiza a necessidade de tratamentos invasivos para controlar a doença. No entanto, surge a questão: qual é o método de diagnóstico mais eficaz?

Entre as abordagens disponíveis, destacam-se o exame de toque retal e o PSA (antígeno prostático específico). Porém, há uma resistência por parte dos homens em relação ao exame de toque retal, frequentemente devido a preconceitos.

Segundo uma pesquisa conduzida pelo Centro de Referência em Saúde do Homem, 20% dos pacientes atendidos recusaram-se a permitir que o médico urologista realizasse o exame de toque retal. Por outro lado, o teste PSA, realizado por meio de exame de sangue, não enfrentou resistência significativa por parte dos pacientes.

A Dra. Fauzia Naime, oncologista do Instituto Paulista de Cancerologia (IPC), especializada em câncer de próstata, afirma que 20% dos diagnósticos da doença são descobertos por meio do exame de toque retal, pois são tumores mais agressivos e não produzem PSA em grande escala.

“O ideal é combinar o toque retal e o PSA, isso possibilita em 80% o diagnóstico. Muitas vezes, quando o tumor é muito pequeno, não é possível diagnosticar por meio do toque retal. Por outro lado, alguns tumores maiores, apesar de serem tocados, podem não expressar PSA. Além disso, exames de imagem são fundamentais”, diz a oncologista do IPC. Entre os exames estão a ressonância multiparamétrica da próstata, ultrassons e também biópsias para confirmar o diagnóstico.

Após o diagnóstico, cada caso deve ser avaliado individualmente antes de optar por um tratamento. Eles podem variar entre radioterapia, hormonioterapia, quimioterapia, prostatectomia radical (cirurgia), entre outros.

Como e em qual idade os homens desenvolvem câncer de próstata?
Ocorrências de câncer de próstata são mais comuns em homens de idade avançada. De acordo com dados do INCA, aproximadamente seis em cada dez casos são identificados em homens com mais de 65 anos, sendo raro antes dos 40 anos. A média de idade no momento do diagnóstico é de 66 anos.

“O paciente tem a diminuição dos jatos urinários e um gotejamento maior após a micção. Às vezes, o homem pode sentir sensação de que a bexiga não está completamente vazia. Além disso, o aumento da frequência urinária, incontinência urinária, dor na pelve em casos mais avançados, sangue na urina e até mesmo nas fezes também podem ser sintomas de um câncer de próstata”, explica a oncologista.

À medida que os homens envelhecem, a capacidade de reparo celular diminui. Isso, em comparação com a juventude, aumenta a probabilidade de um crescimento anormal e desregular das células, o que desempenha um papel fundamental no surgimento de um câncer de próstata. Contudo, a idade não pode ser vista como uma sentença deste câncer, pois existem outros fatores que contribuem para o diagnóstico.

“Em resumo, não existe um método isolado para diagnosticar o câncer de próstata. Deve existir uma combinação de exames, alinhados com métodos de prevenção. Evitar alimentos que podem aumentar a incidência do câncer, como, por exemplo, carne vermelha, leite de vaca, embutidos, frituras, alimentos industrializados, não possuir uma vida sedentária e realizar exames de rotina, principalmente em homens com histórico familiar”, finaliza a Dra. Fauzia.

Dor na região abdominal pode significar varizes pélvicas


Sintomas podem ser confundidos com os de endometriose, mas doenças são bastante distintas

Dores na região da pélvis podem ter vários significados, entre eles, a endometriose e as varizes pélvicas. Apesar de terem sintomas semelhantes, os dois diagnósticos são bem diferentes. A endometriose, doença que atinge cerca de 10% da população feminina em idade reprodutiva, ocorre quando o endométrio cresce para fora do útero, atingindo outros órgãos. Enquanto isso, as varizes pélvicas são caracterizadas pela dilatação das veias na região próxima ao ovário e útero.

No caso das varizes pélvicas, a dilatação das veias leva ao aumento do fluxo sanguíneo na área mencionada, que por consequência leva ao desenvolvimento de dor de intensidade variável, que é tolerável pela manhã, mas à noite tende a se transformar em uma sensação de opressão difícil de controlar. “A doença é pouco conhecida entre as mulheres, por isso, pode ser confundida com outras, como a endometriose. Além disso, há demora no diagnóstico, o que leva a um sofrimento desnecessariamente prolongado”, expõe Patrick Bellelis, colaborador do setor de endometriose do Hospital das Clínicas da Universidade de São Paulo.

A explicação por trás dos altos e baixos de dor está na circulação venosa insuficiente. Quando se está deitado, o sangue que fica estagnado na região pélvica flui para o coração, reduzindo a intensidade da dor. Quando em pé, a massa de sangue se acumula no baixo-ventre, pesando nos tubos venosos da pelve e o desconforto aparece. 

Para detectar o problema é necessário um exame realizado tanto externamente, com a sonda apoiada na pele, quanto endovaginalmente, usando ultrassom. O tratamento é feito com cirurgia e a técnica utilizada para este tipo de distúrbio é altamente avançada e minimamente invasiva. Não envolve cortes ou incisões de qualquer tipo e é realizada em regime ambulatorial.

Já a endometriose ocorre quando os restos da menstruação não são expelidos adequadamente e são liberados nas trompas de falópio ou no interior da pélvis. Isso faz com que o endométrio comece a crescer sobre outros órgãos além do útero, formando manchas de tecido endometrial. Esse tecido se insere e se desenvolve em áreas periféricas como ovários, intestino, bexiga, e, em casos mais incomuns, podendo invadir áreas mais distantes, como o pulmão, fígado e até mesmo o coração.

Entre os principais sintomas da endometriose que afetam a vida da mulher, destacam-se as cólicas de forte intensidade e a dificuldade em engravidar. Observar essas dores é importante para identificar se é o caso de recorrer a um médico. “Cólicas mais intensas, que afetam a rotina, ou com características diferentes das habituais devem ser encaradas como sinal de alerta. Por isso, o acompanhamento médico e uma relação franca com seu ginecologista é tão importante. O diagnóstico precoce é fundamental para devolver a qualidade de vida à mulher com endometriose”, frisa Bellelis. 

Clínica Bellelis - Ginecologia
O ginecologista Patrick Bellelis é Doutor em Ciências Médicas pela Universidade de São Paulo (USP); graduado em medicina pela Faculdade de Medicina do ABC; especialista em Ginecologia e Obstetrícia, Laparoscopia e Histeroscopia pela Federação Brasileira de Ginecologia e Obstetrícia (Febrasgo); além de ser especialista em Endoscopia Ginecológica e Endometriose pelo Hospital das Clínicas da USP. Possui ampla experiência na área de Cirurgia Ginecológica Minimamente Invasiva, atuando principalmente nos seguintes temas: endometriose, mioma, patologias intrauterinas e infertilidade. Fez parte da diretoria da Associação Brasileira de Endometriose e Ginecologia Minimamente Invasiva (SBE) de 2007 a 2022, além de ter integrado a Comissão Especializada de Endometriose da FEBRASGO até 2021. Em 2010, tornou-se médico assistente do setor de Endometriose do Departamento de Obstetrícia e Ginecologia do Hospital das Clínicas da USP; em 2011, tornou-se professor do curso de especialização em Cirurgia Ginecológica Minimamente Invasiva  - pós-graduação lato sensu, do Instituto de Ensino e Pesquisa do Hospital Sírio Libanês; e, desde 2012, é professor do Instituto de Treinamento em Técnicas Minimamente Invasivas e Cirurgia Robótica (IRCAD), do Hospital de Câncer de Barretos.

Taxa de detecção precoce de câncer de próstata é quatro vezes maior com exame de sangue PSA
















Comparação é feita com diagnósticos realizados pelo exame do toque; novembro é marcado por incentivo ao cuidado com a saúde do homem

O método mais amplamente conhecido para a detecção do câncer de próstata é o exame de toque. Ele é empregado por profissionais de saúde para avaliar a glândula prostática em busca de inchaços ou caroços que sejam incomuns no reto e possam indicar câncer. No entanto, um estudo realizado pelo Centro Alemão de Pesquisa do Câncer sugere que outros métodos devem ser incorporados à rotina médica para identificar a doença em suas fases iniciais. De acordo com a pesquisa, o teste PSA, que verifica o nível de antígeno específico da próstata, detectou quatro vezes mais casos de câncer de próstata em homens de 45 anos na comparação com o exame do toque. O PSA é realizado por meio de uma amostra de sangue, que é coletada do paciente em laboratório. 

O ensaio foi feito com 46.495 homens, com idade de 45 anos, inscritos entre 2014 e 2019. Metade dos participantes realizou o teste PSA já aos 45 anos de idade, enquanto a outra metade fez o exame do toque aos 45 anos e realizou o PSA até, no máximo, 50 anos. Os resultados demonstraram que o teste de PSA aos 45 anos detectou quatro vezes mais casos de câncer de próstata. A pesquisa foi conduzida por quatro universidades da Alemanha - Universidade Técnica de Munique, Universidade de Hanôver, Universidade de Heidelberg e Universidade de Düsseldorf. O estudo foi apresentado durante o Congresso Anual da Associação Europeia de Urologia, em Milão, no primeiro semestre deste ano.

“O PSA é, atualmente, a principal recomendação para o rastreio do câncer de próstata. Trata-se de um exame simples, realizado apenas com a coleta de sangue, e proporciona ao médico informações relevantes para o cuidado do paciente. É fundamental que os homens compreendam a importância de solicitar o exame e contar com acompanhamento do urologista”, observa o urologista do Hospital São Marcelino Champagnat, Gino Pigatto Filho.

Quando fazer exames de próstata
O câncer de próstata é o segundo tipo de câncer que mais causa mortes entre o público masculino no país. Segundo o Instituto Nacional do Câncer (INCA), o Brasil deve registrar média de 71,7 mil novos casos por ano, entre 2023 e 2025.

Nesse contexto, o ideal é que os homens estejam atentos à própria saúde e realizem acompanhamento frequente. De acordo com as recomendações da Sociedade Brasileira de Cirurgia Oncológica (SBCO), homens a partir dos 50 anos devem realizar exames anuais com um urologista. Por outro lado, homens pretos ou com parentes de primeiro grau com histórico de câncer de próstata devem iniciar a rotina de acompanhamento aos 45 anos. “A nossa recomendação é o acompanhamento com urologista. A combinação do exame PSA com o exame de toque reduz a probabilidade de falha no diagnóstico precoce para cerca de 5%”, orienta o urologista dos hospitais Universitário Cajuru e São Marcelino Champagnat, Bruno Pimpão.

O médico também ressalta os avanços tecnológicos para rastreamento e identificação de tumores. “A ressonância magnética está ganhando cada vez mais espaço não só no rastreamento, mas principalmente como guia para biópsias prostáticas, que é o exame confirmatório. Após a confirmação, a cintilografia óssea, tomografia e até mesmo o PET-CT podem ser úteis para tomada de decisões e também no estadiamento da doença”, complementa.

A orientação médica com urologista deve ser buscada de forma antecipada em casos de dor ou ardência ao urinar, aumento frequente da necessidade de fazer xixi, dificuldade de esvaziar a bexiga, presença de sangue na urina ou no sêmen e dificuldades de ereção.

Tratamento câncer de próstata
O mês de novembro é dedicado ao aconselhamento e orientação para que os homens estejam mais atentos à saúde. A campanha Novembro Azul enfatiza a importância do diagnóstico precoce do câncer de próstata, uma vez que quanto mais cedo a doença for identificada, maiores são as chances de cura. 

Quando o câncer está localizado, ou seja, limitado à próstata, o procedimento mais comum é a cirurgia para remoção do tumor e/ou radioterapia. Quando a doença avançou dentro da própria região da próstata, pode ser necessária uma combinação da radioterapia com tratamento hormonal. Em casos de metástase, ou seja, quando a doença se espalhou para outros órgãos ou tecidos, a terapia hormonal é a abordagem mais indicada.

Em todo o mundo, são registrados cerca de 1,4 milhão de casos de câncer de próstata atualmente, de acordo com a Agência Internacional para Pesquisa do Câncer da Organização Mundial da Saúde (OMS). Essa doença é responsável por aproximadamente 375 mil mortes a cada ano.

Cuidados para prevenção
Hábitos saudáveis auxiliam na prevenção do câncer, conforme orientações da Organização Mundial da Saúde. Dentre os principais, destaca-se o controle da obesidade e sobrepeso, a moderação no consumo do álcool, frituras e doces, bem como o controle do tabagismo. A prática de exercícios físicos é apontada pelas entidades de saúde como um dos diferenciais para uma vida saudável. Além disso, é preciso gerir o estresse do cotidiano, uma vez que ele é altamente prejudicial para a saúde do organismo.

Zumbido, um problema que atordoa 14% da população mundial


Comunidade médica do país se mobiliza neste mês de novembro para alertar o público sobre as causas associadas ao distúrbio que, além de comprometer a audição, pode ser indicativo de doenças mais graves

Levantamento publicado em 2022 pela revista científica JAMA Neurology aponta que o chamado “zumbido de ouvido” (também chamado de tinnitus ou acúfenos) está presente na vida de aproximadamente 740 milhões de pessoas pelo mundo. Ou seja, 14% da população mundial adulta sofre com esse problema.

No Brasil, são cerca de 28 milhões que fazem parte dessa atordoante estatística, conforme dados da Biblioteca Virtual do Ministério da Saúde. Uma questão, portanto, que desafia toda a sociedade mundial e, claro, merece a nossa atenção.

É por essa razão, a propósito, que, anualmente, a comunidade médica se mobiliza em torno da campanha “Novembro Laranja”. O objetivo é justamente conscientizar a população sobre a realidade preocupante do aumento de problemas do ouvido em todas as idades e motivar mais profissionais da saúde a abraçarem as causas relativas ao zumbido.

“Não se trata propriamente de uma doença, mas de um sintoma que pode ser provocado por inúmeros problemas clínicos, otológicos ou neurológicos. Por isso, o zumbido sempre deve ser investigado. Quando o indivíduo começa a apresentar esse tipo de sintoma, deve procurar um médico para avaliação”, alerta o Dr. José Ricardo Gurgel Testa, otorrinolaringologista do Hospital Paulista - referência em saúde de ouvido, nariz e garganta.

De acordo com o médico, algumas condições que levam ao zumbido podem ter origem no próprio sistema auditivo ou em outros sistemas que afetam o ouvido de forma indireta. A perda da audição é a mais comum e pode ser decorrente da deterioração das células sensoriais do ouvido, problemas que alteram a condução do som, exposição a ruídos intensos ou mesmo à presença de excessiva de cerume nos canais auditivos.

Outras causas associadas são possíveis alterações dos ossículos da audição; doença de Ménière (que causa zumbido, vertigem e perda de audição); e neurinoma do acústico (tumor raro que acomete o nervo auditivo), dentre outros distúrbios relacionados à articulação têmporo-mandibular, metabolismo, sistema cardiovascular e, até mesmo, a quadros de depressão e hábitos de consumo pouco saudáveis.

Diagnóstico e tratamentos
O diagnóstico, segundo o especialista, depende da avaliação do tipo de zumbido (ou seja, qual a emissão sonora que é captada); quando surge; o tempo que dura; além dos sintomas associados, que eventualmente podem incluir tontura, desequilíbrio ou palpitações, por exemplo.

Um dos exames mais recomendados é a acufenometria, que consiste na emissão de sons com determinadas frequências até o paciente identificar um som semelhante ao zumbido que o incomoda. O procedimento é realizado a partir da avaliação de um fonoaudiólogo ou otorrinolaringologista, com o audiômetro – equipamento utilizado para a obtenção dos limiares auditivos.

Esse trabalho de investigação, ainda segundo o médico, também inclui a observação interna dos ouvidos, mandíbula e vasos sanguíneos da região, além de exames de imagem, como tomografia computadorizada ou ressonância magnética, que podem identificar de forma mais precisa alterações cerebrais ou na estrutura dos ouvidos.

“Para tratar o zumbido, é necessário conhecer a sua causa. Algumas vezes, o tratamento é simples, podendo incluir a remoção de cera pelo médico otorrinolaringologista, o uso de antibióticos para tratar a infeção ou uma cirurgia para corrigir defeitos no ouvido, por exemplo”, afirma. 

Entretanto, em alguns casos, conforme o Dr. Testa, o tratamento é demorado e mais complicado, podendo necessitar de um conjunto de terapias que podem ajudar a aliviar os sintomas ou a diminuir a percepção do zumbido. “Por isso, é importante que as pessoas tenham consciência sobre o tema e investiguem as causas”, finaliza.

Sobre o Hospital Paulista de Otorrinolaringologia
Fundado em 1974, o Hospital Paulista de Otorrinolaringologia possui quase cinco décadas de tradição no atendimento especializado em ouvido, nariz e garganta e durante sua trajetória, ampliou sua competência para outros segmentos, com destaque para Fonoaudiologia, Alergia Respiratória e Imunologia, Distúrbios do Sono, procedimentos para Cirurgia Cérvico-Facial, bem como Buco Maxilo Facial.

Referência em seu segmento e com alta resolutividade, conta com um completo Centro de Medicina Diagnóstica em Otorrinolaringologia, assim como um Ambulatório de Olfato e Paladar, especializado no diagnóstico e tratamento de pacientes com perda total ou parcial dos sentidos. Dispõe de profissionais de alta capacidade oferecendo excelentes condições de suporte especializado 24 horas por dia.

Em vez de descongestionante, que tal optar pela lavagem nasal?


Além de não oferecer riscos à saúde, procedimento também é eficaz para desentupir as narinas e ainda serve como método preventivo a crises de rinite alérgica; médica do Hospital Paulista ensina o passo a passo

Todo mundo sabe que o uso recorrente de descongestionantes nasais, além de viciar, causa efeitos colaterais bastante danosos, especialmente ao coração.

“Mas como viver sem eles, se o meu nariz vive entupido?”. Essa é a pergunta que a maioria das pessoas faz aos médicos quando alertadas sobre a necessidade de evitar o uso desse tipo de medicamento, principalmente quem tem rinite alérgica, justamente pela ocorrência mais frequente de obstrução nasal.

A resposta nem sempre agrada, mas não resta dúvida que a lavagem nasal é a opção mais segura e eficaz para eliminar esse desconforto – seja em bebês, crianças, adultos ou idosos.

“Embora um pouco mais trabalhosa de se fazer, a lavagem nasal não causa riscos à saúde e pode ser até mais eficaz em termos de resultados, já que não provoca o chamado ‘efeito rebote’, típico dos descongestionantes. Isto é, quando o efeito passa, o nariz volta a entupir, até com mais intensidade. Com a lavagem, isso não ocorre”, explica a Dra. Cristiane Passos Dias Levy, otorrinolaringologista do Hospital Paulista – referência em saúde de ouvido, nariz e garganta.

O procedimento consiste na introdução de uma solução salina suave e estéril nas narinas para ajudar a limpar o muco, aliviar a congestão nasal e reduzir a inflamação dentro do nariz. A dosagem recomendada varia conforme a idade e deve ser aplicada de acordo com a seguinte escala: 1ml (para bebês de 0 a 6 meses); 3ml (para crianças de 6 meses a 2 anos); 3ml a 20ml (2 a 6 anos) e 10ml a 20ml (a partir de 7 anos).

O ideal, segundo a especialista, é utilizar o soro fisiológico comprado pronto na farmácia com a ajuda de um aplicador (parecido com um conta-gotas). Mas também é possível fazer uma mistura caseira em casos de emergência: um litro de água que tenha sido fervida com, aproximadamente, uma colher de chá de sal.

“Além de melhorar a respiração nos momentos de crise, a lavagem também serve para prevenir infecções sinusais e reduzir a frequência de episódios de sinusite em pessoas com rinite. Portanto, pode ser feita regularmente”, enfatiza a médica.

Como fazer?
A frequência da lavagem nasal pode ser de duas a três vezes ao dia. É recomendado usar de 10-20 ml de solução salina por narina na seringa. Se for um jato contínuo, de 3 a 10 segundos. Se for na garrafinha, use uma específica para esse fim, de 120 a 240 ml, mantendo a proporção de 10-20 ml de solução salina por narina. Seguidas tais recomendações, os próximos passos são:

1. Inclinar o dorso levemente para a frente (cerca de 15 graus);

2. Abrir a boca e iniciar uma respiração oral. Essa manobra irá realizar uma elevação do céu da boca, protegendo os ouvidos, impedindo dor na região, devido à pressão da solução no tubo auditivo (canal que liga a parte de trás do nariz ao ouvido).

3. Colocar a ponta da seringa, ou de outro material que for usado, sem agulha, na narina, realizando uma boa vedação. Evite que a seringa machuque o septo e a concha nasal inferior, posicionando adequadamente a ponta da seringa.

4. Após vedar a narina, inclinar lateralmente a cabeça, em um ângulo de mais ou menos 30 graus para o lado contrário de onde está o material usado, e esguichar a solução. Não se esqueça de realizar a lavagem nasal com movimentos suaves para evitar lesões traumáticas.

De acordo com a Dra. Cristiane, é recomendável repetir esse procedimento de 3 a 4 vezes em cada narina, dependendo da necessidade. Além disso, pode-se encher a seringa com mais quantidade de soro, já que será eliminado pela outra narina.

Sobre o Hospital Paulista de Otorrinolaringologia
Fundado em 1974, o Hospital Paulista de Otorrinolaringologia possui quase cinco décadas de tradição no atendimento especializado em ouvido, nariz e garganta e durante sua trajetória, ampliou sua competência para outros segmentos, com destaque para Fonoaudiologia, Alergia Respiratória e Imunologia, Distúrbios do Sono, procedimentos para Cirurgia Cérvico-Facial, bem como Buco Maxilo Facial.


Exame Covid-AG com resultado em
até 1 hora no Laboratório AnaLab

Com aumento do número de casos da COVID19, sabemos da importância de um diagnóstico rápido e preciso.

No Laboratório AnaLab você realiza seu exame durante todo nosso horário de atendimento com resultado em até 1 hora.

Somos o único laboratório realizando os testes para Covid19 e Influenza.

Se você estiver com sintomas ou contato com paciente positivo, realize seu exame, o diagnóstico é a única forma de conter o avanço da pandemia.

Contate nosso setor de atendimento

(15) 3542-4159

(15) 99797-4159




Ribeirão Grande inicia vacinação contra a covid aos adolescentes com comorbidades

Jovem do Ferreira dos Matos foi o primeiro a receber a primeira dose da vacina Pfizer

A Prefeitura Municipal de Ribeirão Grande, através da diretoria de Saúde, iniciou nesta última terça-feira, dia 17, uma nova etapa do seu plano de imunização contra a Covid-19 com a aplicação da primeira dose da vacina aos adolescentes de 12 a 17 anos que possuem comorbidades, grávidas e puérperas.

Feliz da vida, o adolescente Pedro Henrique Jacinto Ferreira, de 13 anos, morador do bairro Ferreira dos Matos, foi o primeiro a receber o imunizante da Pfizer. Sua mãe, Priscila Daniele Ferreira, de 37 anos, comemorou a vacinação. “A gente fica meio assustada por ele ter comorbidade, mas ao mesmo tempo, ficamos tranquila por ter recebido a vacina”, disse.

A vacinação está ocorrendo no Posto de Saúde da cidade, das 08h00 às 16h00, e vai até o dia 25 de agosto, segundo o Governo de São Paulo, responsável pela entrega das vacinas aos municípios paulistas. Para os adolescentes de 12 a 17 anos sem comorbidades, a vacinação deve começar a partir do dia 30 de agosto.

Os adolescentes deverão apresentar documentos como cartão SUS, RG, CPF, comprovante médico da comorbidade e com acompanhamento de um responsável.

Para o prefeito Marcelo Nunes, a imunização destes jovens tem também como objetivo dar maior segurança às famílias para o retorno das aulas presenciais. “É, sem dúvida, mais um passo importante no enfrentamento à Covid-19, para a segurança das famílias e o retorno das aulas presenciais”, comentou.

Ribeirão Grande inicia vacinação de jovens a partir de 18 anos

Cidade foi a primeira da microrregião de Capão Bonito
a vacinar essa faixa etária

A Prefeitura Municipal de Ribeirão Grande, através da Diretoria de Saúde, iniciou nesta segunda-feira, dia 9, a vacinação para os jovens acima de 18 anos. A imunização começou às 13h00 na Unidade Básica de Saúde do município, e encerrou às 19h00.

De acordo com o diretor de Saúde, Miguel Ferreira, até as 17h00, cerca de 250 jovens já haviam recebido a primeira dose da imunização. Ao todo, a cidade recebeu do Centro de Vigilância Sanitária do Estado de São Paulo, 422 doses da vacina contra a Covid-19.

O prefeito Marcelo Nunes também foi decisivo na antecipação da vacinação para essa faixa etária na cidade. “Para seguirmos a abertura mais ampla dos setores econômicos proposta pelo Governo de São Paulo, entendemos a vacinação como primordial para garantirmos a saúde de todos os ribeirãograndenses. Apesar de recebermos um pequeno lote de vacinas, tivemos a coragem de antecipar a imunização aos jovens acima de 18 anos”, comentou.

A cidade de Ribeirão Grande virou referência na região pelas políticas públicas adotadas no enfrentamento da Covid-19. O município, sob o comando do prefeito Marcelo Nunes e do vice-prefeito Ricardo de Lima, conseguiu disponibilizar um auxílio financeiro no valor de R$ 1.000,00 aos comerciantes e gratificação salarial a todos os servidores municipais que atuaram na linha de frente.

Amanhã é feriado e o Laboratório AnaLab
estará ABERTO para atendimento
exclusivo de exames COVID-19



Horário de funcionamento: 7h-13h. ⁣

Pacientes com suspeita de covid-19 entre
em contato conosco e agende o seu exame. ⁣

Caso possua dúvidas, contate nosso setor
de atendimento ao cliente: ⁣⁣

📍UNIDADE I - CAPÃO BONITO⁣⁣⁣

Rua Silva Jardim, 597 – Centro⁣⁣⁣

Telefones (15) 3542-4159 / (15) 99797-4159 ⁣⁣⁣

⁣⁣

📍UNIDADE II - RIBEIRÃO GRANDE⁣⁣⁣

Rua João Batista Brisola, 47 - Centro⁣⁣

Telefones (15) 3544-6778 / (15) 99642-3336

⁣⁣

AnaLab, nossa especialidade é

cuidar de quem você ama.

Covid-19: nova linhagem do vírus
é identificada no interior paulista

Variante tem circulado na região das cidades
paulistas próximas da divisa com MG

Foi confirmada pela Sociedade Brasileira de Virologia (SBV), nesta terça-feira (25), a identificação de uma nova linhagem brasileira de coronavírus, a chamada P4, que tem circulado em algumas cidades paulistas.

O estudo de identificação, fomentado pelo Ministério da Ciência, Tecnologia e Inovação, mostrou que a nova linhagem apresenta a mutação L452R na proteína do vírus. Segundo a SBV, a nova variante tem circulado na região das cidades paulistas próximas da divisa com Minas Gerais, como Mococa, Caconde e Itapira.

A P4 também tem circulado na região de Porto Ferreira, Descalvado, Itirapina, Capão Bonito, São Miguel Arcanjo, Itapetininga, Iperó e Cesário Lange. Ainda não há informações se a nova linhagem é mais transmissível ou mais letal que as demais.


Exame toxicológico no Laboratório AnaLab


Atenção motoristas! Desde o dia 3 de maio, o Laboratório AnaLab voltou a realizar o exame toxicológico.

Lembrando que a obrigatoriedade desse exame é para os motoristas da categoria C, D e E.

A não regularização do exame, pode resultar em multa, pontos na carteira e suspensão da mesma por meses. 

Agende seu exame conosco:

📍UNIDADE I - CAPÃO BONITO⁣
Rua Silva Jardim, 597 – Centro⁣
📞(15) 3542-4159/(15) 99797-4159⁣
📍UNIDADE II - RIBEIRÃO GRANDE⁣
Rua João Batista Brisola, 47 - Centro⁣
📞(15) 3544-6778 / (15) 99642-3336.⁣

AnaLab, nossa especialidade
é cuidar de quem você ama!





O Laboratório AnaLab informa que está realizando o exame de sorologia para Coronavírus (COVID-19).

Esse exame deve ser realizado por pacientes que possuam sintomas ou que tenham entrado em contato com alguém infectado há pelo menos 7 dias.

Lembrando que a avaliação do resultado deve ser feita por seu médico.

O exame deverá ser agendado previamente pelos telefones.

📍UNIDADE I - CAPÃO BONITO
Rua Silva Jardim, 597 – Centro
Telefones (15) 3542-4159 / (15) 99797-4159 

AnaLab, nossa especialidade
é cuidar de quem você ama!



Atchimm!




Cerca de 70% dos dentistas sofrem
com problemas nos membros superiores

Lesões por Esforços Repetitivos afetam, principalmente,
as mãos; SBCM orienta para prevenção

Nas mãos do dentista estão a entrega de um sorriso mais saudável e bonito, mas também, estão dores. Isso porque esses profissionais são comumente acometidos pelas LERs/Dort, doenças caracterizadas por dor crônica que comumente afetam músculos e nervos e membros superiores, e cuja etiologia está relacionada aos movimentos repetitivos realizados no trabalho. Segundo o CROSP (Conselho Regional de Odontologia do Estado de São Paulo), cerca de 70% dos profissionais da área relatam algum tipo de dor pelo corpo, principalmente nos membros superiores.

“O dentista emprega força em muitas ações realizadas durante os tratamentos, necessitando ainda de precisão e execução de movimentos finos em grande parte dos procedimentos. Essa situação aumenta o risco desses profissionais desenvolverem variadas patologias”, explica o presidente da SBCM (Sociedade Brasileira de Cirurgia da Mão), Dr. João Baptista Gomes dos Santos. A entidade lança campanha, em parceria com o CROSP, de conscientização aos dentistas, com orientações para evitarem problemas.

Lesões mais frequentes
Síndrome do túnel do carpo: trata-se da compressão do nervo mediano em nível de punho. “Além da dor, esse problema traz perda de sensibilidade e impotência funcional durante a flexão do primeiro, segundo e terceiro dedos”, explica o especialista, completando que essa síndrome está mais ligada à repetição do movimento do que à força empregada.

Síndrome do túnel ulnar: atinge a face flexora e extensora do quarto e quinto dedos e região hipotenar. Provoca dor, alterações na sensibilidade, perda de força e impotência funcional atingindo a face ulnar da mão.

Tendinites: inflamações que acometem os tendões. “Uma das doenças mais comuns é a síndrome de De Quervain, tendinite que afeta o polegar”, pontua o presidente da SBCM.
Tenossinovites: são inflamações das bainhas tendinosas e que, geralmente, acometem os músculos flexores ou extensores do punho e dedos, causando dor e dificuldades de realizar movimentos. “Nesse problema, há a presença de edema e perda de força muscular”, salienta o Dr. João.

Diagnóstico e tratamento
O presidente da SBCM explica que o diagnóstico das LERs/Dort é basicamente clínico, no qual é feita uma análise da vida profissional pregressa, da história da doença e de exame físico minucioso. “O médico irá analisar o tipo de função realizada no trabalho, a frequência dos movimentos, os equipamentos empregados, o tempo na função e a existência de pausas durante o trabalho”, lista.

O tratamento para as LERs/Dort vai desde o uso de medicação, fisioterapia e exercícios físicos, até cirurgias em casos mais avançados. “Nas fases iniciais, os sintomas das LERs/Dort são mais leves, o que possibilita tratamentos mais simples. Caso a pessoa não dê muita atenção logo no início, isso pode levar a evolução da doença a estágios mais severos, onde os sintomas são mais acentuados, as dores permanentes e com maior dificuldade de cura” ressalta o médico.

Prevenção
Exercícios de alongamento nas mãos são importantes para prevenir as LERs/Dort. “Além de reduzir as tensões musculares, a prática previne lesões”, destaca o Dr. João.

Realização de pausas durante a jornada diária também são fundamentais para manter o exercício da profissão sem transtornos.

Sobre a Sociedade Brasileira de Cirurgia de Mão
A SBCM (Sociedade Brasileira de Cirurgia de Mão), fundada em 1959, congrega médicos especialistas em Cirurgia da Mão e Reconstrutiva do Membro Superior. A instituição promove a formação de profissionais, além de fornecer condições para atualização permanente, sob a forma de ensino, pesquisa, educação continuada, desenvolvimento cultural e defesa profissional.
Mais informações em http://www.cirurgiadamao.org.br/

Automedicação é um perigo
para a saúde bucal

Uso autoprescrito de medicamentos dificulta tratamento e pode agravar doenças


Todos conhecemos, ou até mesmo somos, aqueles que montam uma farmácia particular em casa com uma variedade de remédios para dores musculares, dores de cabeça, coriza, febre, indigestão, entre outros sintomas que podem nos acometer no dia-a-dia. Mas, não atentamos para o fato de que a automedicação pode se tornar um problema sério quando vira rotina. 

Dados do Sistema Nacional de Informações Tóxico-Farmacológicas (SINITOX), mostram que os medicamentos ocupam o primeiro lugar entre os agentes causadores de intoxicações em seres humanos e o segundo lugar nos registros de mortes por intoxicação. O Sistema também aponta que apenas 50% dos pacientes, em média, tomam seus remédios corretamente, ou seja, cumprem a duração e intervalo de uso de medicamentos estipulados por profissionais da saúde.  

Além de mascarar sintomas recorrentes que podem indicar alguma doença grave, o uso de medicamentos autoprescritos pode ser nocivo para a saúde bucal. De acordo com cirurgião-dentista Sidney das Neves, integrante da Câmara Técnica de Cirurgia e Traumatologia Bucomaxilofacial do Conselho Regional de Odontologia de São Paulo (CROSP), a utilização indiscriminada de antibióticos e antiinflamatórios é a mais preocupante. 

Segundo ele, a automedicação para tratar uma infecção na região da boca pode resultar em graves consequências para a saúde geral. “Os processos infecciosos na região maxilofacial e cervical evoluem de maneira muito rápida, por isso, o tratamento sem um acompanhamento do cirurgião-dentista pode resultar em uma terapia ineficaz, podendo até mesmo contribuir para o agravo da condição”, informa Neves. 

Usar esses medicamentos sem prescrição de um profissional qualificado também pode alterar os efeitos do uso de outro remédio que o paciente esteja utilizando, diminuindo ou anulando completamente o resultado terapêutico desejado. “Especificamente no caso dos antibióticos, o uso inadvertido, ao invés de combater de forma adequada o micro-organismo responsável pelo processo infeccioso bucal, por exemplo, poderá auxiliar na seleção de uma bactéria muito mais resistente, levando risco à pessoa”, afirma o cirurgião-dentista.

Não só a população, mas os próprios profissionais da saúde devem estar atentos ao receitar antimicrobianos. Cirurgiões-dentistas e médicos devem seguir as determinações da Organização Mundial da Saúde (OMS), que aconselha a indicação de medicamentos que ofereçam o máximo de efeito terapêutico com o mínimo de toxicidade e de potencial de desenvolvimento de resistência microbiana. 

Bisfosfonatos e os problemas maxilares
Outro tipo de medicamentos que podem vir a causar problemas para a saúde bucal são os bisfosfonatos. Os bisfosfonatos são uma classe de medicamentos que previnem a diminuição da densidade mineral óssea, geralmente usados no tratamento de osteoporose e outras doenças que causam fragilidade dos ossos, como alguns cânceres.  

Apesar de seus benefícios, esta classe de remédios coloca seus usuários em risco de uma doença dos maxilares chamada de osteonecrose bisfosfonada. A doença provoca a morte do osso pelo corte do suprimento de sangue, causando dor e dificuldade de movimento da mandíbula.  

O maior risco de desenvolvimento da osteonecrose no maxilar é a presença de focos infecciosos na gengiva ou dente de pacientes que recebem estas medicações. Por isso, é aconselhado as pessoas que realizam tratamentos com bisfosfonatos (alendronato, ibandronato, risedronato, pamidronato, clodronato e ácido zoledrônico), procurarem um cirurgião-dentista experiente ou um cirurgião bucomaxilofacial para acompanhamento e controle da saúde bucal.  

Sobre o CROSP/ – O Conselho Regional de Odontologia de São Paulo (CROSP) é uma autarquia federal dotada de personalidade jurídica e de direito público com a finalidade de fiscalizar e supervisionar a ética profissional em todo o Estado de São Paulo, cabendo-lhe zelar pelo perfeito desempenho ético da Odontologia e pelo prestígio e bom conceito da profissão e dos que a exercem legalmente. Hoje, o CROSP conta com mais de 130 mil profissionais inscritos. Além dos cirurgiões-dentistas, o CROSP detém competência também para fiscalizar o exercício profissional e a conduta ética dos Técnicos em Prótese Dentária, Técnicos em Saúde Bucal, Auxiliares em Saúde Bucal e Auxiliares em Prótese Dentária. Mais informações: / www.crosp.org.br

Nenhum comentário:

Postar um comentário